Você está na página 1de 73

CASE-BASED

TEACHING NOTES
MEDICINE CBT
DATE/ TIME CBT TOPIC LECTURER DATE/ TIME CBT TOPIC LECTURER
Sep 1st 1. Exam Technique Dr. Spooner Sept 4th 3.MS/peripheral Dr. Gargoum
neuropathy
10:00-12:00 2. Movement Disorders Dr. Spooner 10:00-12:00 4. Headache Dr. Geoghegan

Sep 7th 5.Cognitive decline Dr. Gogarty Sept 11th 7. Thyroid dx Dr. Sui
09:30-11:30 6.Bone Health Dr. Geoghegan 10:00-12:00 8. Diabetes Dr. Omar
mellitus
Sep 14th 9. Cough Dr. James Sept 18th 11. Exam Dr. Spooner
Technique 2
09:30-11:30 10. Wheeze Dr. Landers 10:00-12:00 12. Murmur Dr. James

Sep 21st 13. Dsypnoea Dr. Landers Sept 25th 15. Chest pain Dr. Spooner

09:30-11:30 14. Palpitations Dr. Geoghegan 10:00-12:00 16. Chest pain Dr. Spooner

Sep 28th 17. Fever in Dr. Landers Oct 2nd 19.Back Pain Dr. Spooner
Neutropenic Patient
09:30-11:30 18. Approach to skin Dr. Boselli 10:00-12:00 20. Acute swollen Dr. Gogarty
lesions joint
Oct 5th 21. Pyrexia of Unknown Dr. Jagtap Oct 9th 23. Dr. Mc Laughlin
Origin Bleeding/clotting
disorders
09:30-11:30 22. Connective Tissue Dr. Quinn 10:00-12:00 24. Anaemia Dr. James
Disease
Oct 12th 25. IBD Dr. Mc Laughlin Oct 16th 27. Jaundice Dr. Spooner

09:30-11:30 26. Malabsorption Dr. Omar 10:00-12:00 28. Connective Dr. Spooner
tissue disease
Oct 19th 29. Care of the Palliative Dr. Quinn Oct 23rd
patient
09:30-11:30 30. Oliguria Dr. Thebe No session
/Proteinuria scheduled
Tuesday 1/9/2015
2.MOVEMENT DISORDERS

APPROACH
Dyskinesias
Rhythmic
o Tremor
Non-rhythmic
SLOW SUSTAINED RAPID
Athetosis Dystonia Suppressible Non-suppressible
Snake- Contractio Tics Chorea (restless)
like n or Parakinesias Myoclonus (rapid ctraction)
Writhing spasms o voluntary suppression when Ballismus (always start proximally, large
Painful almost fallen etc movements amplitude)
Sustained Hemiballismus (always on one side)
abN
movements

CASES
1. A 33 year old man presents to the Neurology out-patient as a referral from his general practitioner. He has a six
month history of worsening ataxia. In association, there is weakness in all limbs- he feels a dragging sensation
when walking and frequently drops items, or mis-focusses when going to grip something. (+dyscoordination)

Ataxia: often assocd with chorea


DDX
Friedrichs ataxia: most common, young, hereditary, progressive
MS: in cerebellum, usually local lesions not as various as in case (GLOBAL weakness), no optic neuritis
SACDS: peripheral neuropathy (weakness but with sensory loss wh is usually dominates), in older, malnourished, >
likely to have proprioceptive VS cerebellar sensory loss dorsal column involved
HDD
Ataxia telangiectasia: in chr 11 ATM gene depending on anticipation and penetrance (incomplete)
Wilsons
Infection: HIV
Neoplasm: in basal ganglia
Spino-cerebellar ataxia: hypertonic, usually do not have weakness in ALL 4 limbs
Dopamine agonists
Amphetamine

2. A 33 year old man presents to the Neurology out-patient as a referral from his general practitioner. He has a
three month history of involuntary movements of his limbs They are unpredictable and usually involve his
hands, but sometimes spread to feet and face.

Chorea
DDX
Seizure- myoclonic epilepsy
Chorea
Ballismus
Dystonia

3. A 33 year old man presents to the Neurology out-patient as a referral from his general practitioner. He
complains of episodes of muscle contraction and spasm which can be painful

Dystonia
DDX
Dystonic disorder
Wilsons: young
Myoclonus: not painful
Hypocalcaemia: tetani
INVESTIGATIONS
Hx
o HPC
timeline, onset
acute: trauma, meds
chronic: congenital
pregnancy, if on COCP or not
HDD (chr 4) Friedrichs ataxia (chr 9) Ataxia telangiectasia (chr 11)
Sustained behavioural changes Fine motor (dysphagia & dysarthria) Eye movement disorder
Cognitive decline & sensory abN (dorsal column- DM
Psychiatric disturbances proprio & light touch) Immune deficiency (presents w
Motor impersistence (differentiate Cardiomyopathy (angina, features of recurrent infection particularly
from chorea) CCF if late, exertional dyspnoea) & respiratory & haematological
diabetes malignancies
Kyphoscoliosis (can get restrictive
lung dzs)
o PMH
malignancy: cerebellar paraneoplastic syndrome (ovarian mostly), primary or metastatic
o FHx
death at early age of unknown cause

Bloods
o FBC
lymphocytosis, IgA, IgE (low levels in telangiectasia), ceruloplasmin & Cu
o Full septic , toxicology for amphetamine, metabolic screen
o Pregnancy test
Other
o LP-oligoclonal bands
o EMG: MS
o EEG: spikes in myoclonic disorders
o T2 weighted MRI of brain & spine
o Genetic testing

MX
Chorea of unknown aetiology
o no cure
o MDT monitoring
physios for movements & fall risk minimisation + balance, with OT helps
speech & language & eating n swallowing (aspiration pneu)
nutrition for DM, enteral feeding (PEG tube feed)
specialists
o pharmacological
anti-cholinesterase for resting tremor
B-blocker for essential tremor

if 2y: tx underlying causes


o antimicrobials for infection
o decompressive surgery if trauma
o debulking therapy or radiotherapy if malignancy
Friday, 4/9/2015
3.MS/PERIPHERAL NEUROPATHY
CASES
1. A 73 year old man presents to the Neurology clinic with a 6 month history of progressively worsening
parasthesiae of the lower limbs. This is associated with occasional darting pain in the feet. On examination, there
is reduced sensation to vibration and light touch in a non-dermatomal distribution.
DEFINE
LHermittess sign: barbers chair sign electric shock down into arms & legs
Uhtoffs phenomenon: increase in temp leads to exaggerration of CNS symptoms (exercise, weather, hot shower)
Demyelination: damage to myelin sheath iinsulation interferes cduction
Axonal neuropathy: degeneration of ENTIRE nerve
Spacticity: altered skeletal musc pformance: combi of paralysis & power, increased tendon reflexes & hypotonia (look
rigid, stiff & contracted)
CMT: motor & sensory neuropathy, damage to arms & legs, peri nerves musc weakness & wasting (loss of muscles
bulk), old pts
Oligoclonal bands: Ig present in CSF, absent in blood

DDX
Peripheral neuropathy 2y to diabetes non-dermatomal, progressive, peripheral
transverse myelitis pain, preceding viral infections
lumbar radiculopathy trauma? Disc herniation hx? ve: usually one-sided
mononeuritis multiplex one nerve affected, affecting multiple sites
o ex of other mononeuropathy: carpal tunnel syndrome, sciatica
B12 deficiency
GB
CMT: inherited progressive loss of musc tissue & light touch

INVESTIGATION
Hx & exam
FBC: fasting glucose if borderline OGTT + HbA1c, WCC, Vit B12 & folate (defy)
U&E: electrolytes imbalance
TFTs: hypothyroidism
Toxicology screen: alcohol, heavy metals
Vasculitic screen: ANCA, ANA, CRP, ESR
Nerve conduction studies:

Classify neuropathy
Pulses, doppler US to investigate PVDs

Dorsal column proprioception, light touch, vibration


Spinothalamic crude touch, pain, temp

OTHER QXs
MEDS
Hearing loss, muscle atrophy (vascular aetiology), unintentional weight loss (malignancy)

MX
Diabetic
MDT mx
Optimise glycaemic control
Physio- manage fall risk,balance, orthoses, shoes
OT
Transverse Myelitis
Tx neuropathic pain: amytriptylin, gabapentin, pregabalin VS MSK pain
B12 deficiencies: injections
Alcohol
B Vitamins Ivly (Pabrinex) over 3days, detox
Infections: Ab
Toxins
heavy metals chelating agents
Hypothyroidism: Thyroxine
Vasculitis
Steroids, cyclophosphamides (IMMUNOSUPPRESSANTS)
2. A 29 year old woman presents to the emergency department with right leg weakness. On examination right leg
power was 3/5, right leg reflexes were brisk, and the right plantar reflex was up-going. She has no past medical
history and has never attended hospital before

DDX
MS: age gp, UMN signs (weakness, reflex, upgoing plantar reflex)
Stroke L hemisphere
SOL: headaches, N&V, weigh loss, blurred vision, cfusion
ALS

INVESTIGATION
Hx & exam
Diagnostic
o MRI brain & spinal cord: demyelinating plaques disseminated at different times and places, Gadolinium enhanced &
T2-weighted shows up demyelination well
o CSF: oligoclonal bands, WCC, protein send for electrophoresis, r/o infection,cerebral vasculitis, neurosarcoid,
cytology if malignancy suspected
o Visual evoked potentials: slow in MS (loss of myelin impair transmission)
To r/o other aetiology
o Septic screen

KEY CLINICAL FEATURES


Sensory impairment?
Timeline: acute >worrying (stroke/ malignancy) VS indolent/ chronic
Hx of autoimmune disease

3. A 29 year old woman presents to the emergency department with a right leg weakness. On examination right leg
power was 3/5, right leg reflexes were brisk, and the right plantar reflex was up-going. She reports an episode of
blurred vision three months ago lasting three weeks

DDX
MS: +optic neuritis
Stroke
SOL
Neurosarcoid
Cerebal vasculitis

MX
Acute flare:
Optic neuritis
High dose IV Csteroids (methylprednisolone) 3-7days + oral streoids & taper thereafter + PPI! + biphosphonates
Neuropathic pain
Classify type of MS
RR & 2y progressive
Pharmacological
Interferon subcutly natalizumab monthly (also given to IBD) if not responding (usually 2y progressive)
Interferon reduces freq of relapse, but cause myalgia-like illness
Immunosuppressants cyclophosphamide
MDT
4.HEADACHE
CASES
1. A 33 year old man presents to the Emergency Department complaining of severe occipital headache. He is
normally fit and healthy and takes no regular medications. His symptoms started suddenly 2 hours ago while
lifting weights in the gym and were associated with vomiting.

DDX
SAH: young, severe headche, no hx of headache VS SUBDURAL fall on head VS EXTRADURAL
Raised ICP fr burst berry aneurysm
SOL (behavioural change, weight loss, early morning vomiting?)
Dissection of vertebral artery hyperextension of neck in RTA, hairdressers
AVM
tension-typed headache (bifrontal)
meningitis (no fever, rash comes late though!)
migraine
vasculitis

INVESTIGATIONS
Diagnostic
CT brain: bleeding (1/2 hr after onset at least)
CTangio: highlight leakage points
LP

CLINICAL FEATURES
SAH
o Bradycardic/ initial tachycardic response, psistent & severe hypertension (do not drop BP too quickly! CVP >16,
BP>140/60), GCS, worst headache, miserable, vomiting
Exam: fixed dilated pupils, hemiparesis, papiloedema, seizure

TX/MX
ABC
Monitor vitals & IV access 2 large bore cannulas
Call neurosurgeons
BP control: titrate IV B-Blockers (labetalol) maintain <140-160, anything >180/100 red alert!
o if CI to B-blockers may use GTN patch, Levodopa, hydralazine, mannitol

CVP monitor
IV paracetamol +/- tramadol. Not codeine-consisting drugs worsen headache & NSAIDs bleeding
Anti-emetics: metaclopramide, cyclizine (SE: sedating)
Anti-pyrexics
Check if theres any urinary retention
Stop anticoags if on any
Seizures lorazepam, phenytoins if benzo CId

COMPLICATIONS
Rebleed
drop in GCS, hypotension, bradycardia, hiccupping, seizures
Mx: re-scan
Aneurysms
Mx: clipping(neurosgery) or coiling (IR)
SAH
death, hydroceph (do VP shunts)
2. A 20 year old man presents to his GP complaining of headache and neck stiffness. His symptoms came on
gradually over the last day. In association he is febrile at 38.7C and has a rash.

DDX
Meningococcal meningitis VS viral( + flu-like, subacute, low grade fever) VS fungal VS parasitic
o RF: immunosuppressed, asplenism, elderly, pregnancy
o cause: N.meningitidis, H.i, strep pneu, Listeria, HSimplex
Encephalitis cognitive impairment, sustained bhvrl changes
SAH

CLINICAL FEATURES
Photophobia, neck stiffness, Kernigs sign, Bradzunskis sign, focal neurological signs

INVESTIGATIONS
Bloods: Coag, INR>50 , pL can x LP if abN
Imaging: r/o SOL, abscess [RULE: CT before LP!]
LP
Ab: cefuroxime (3rd gen ceph) + vancomycin (if severe) + amoxicillin (if elderly), benzylpenicillin (community settings)
MRI: encephalitis (high signals highlighted)

MX
Viral: antipyretics, antivirals (usually wait until PCR result is out-aciclovir)
Bacterial:
***WCC differentiation of LP to know bac VS viral

3. A 67 year old man presents to his GP complaining of new left sided headache. He has a past medical history of
hypertension and osteoarthritis and he takes amlodipine 5mg daily and paracetamol on a PRN basis. He also
reports pain in his jaw while chewing.

DDX
Giant cell /temporal arteritis
o Complication: visual loss (via retinal arterial occclusion or optic nerve atrophy)
o no specific timeline
o RF:Polymyalgia rheumatica (50% association) (musc pain on prox jts + tender jts- limited mobility, falls)
Medication-induced headache amilodipine
TMJ syndrome
Trigeminal neuralgia
Horrible tooth ache
Trauma
Migraine
Postural hypotension
SOL

INVESTIGATIONS
Bloods
ESR raised temporal artery biopsy if ve: *might miss site of arteritis (usually begins down in carotid)
Imaging
*do US

TX/MX
GCTA: high dose oral / IV steroids prednisolone (60mg) + biphosponates + , low-dose aspirin (reduce risk of visual loss
& stroke), analgesics. Cyclophos / methotrexate if CId to steroids
TMJ syndrome
Trigeminal neuralgia
Neuropathic pain: Carbamazepine, pregabalin, gabapentin
Microvasc decompression if compression aetiology

CLINICAL FEATURES
scalp tenderness, severe headache, diminished pulses, fever
Monday, 7/9/2015
5.COGNITIVE DECLINE

DEFINITION
Mild cognitive impairment: memory impairment not yet impacting on fxal ability
Delirium: acute change in cognition & confusional state, usually organic in cause, fluctuating course
Dementia: chronic & progressive, decline in cognitive ability in more than one of memory, etc with lost of fxal ability

KEY CLINICAL FEATURES


Nature of memory prob Timeline, examples of incidents
Level of fx
Contributing factors level of education, drinking, trauma
Associated symptoms depression
Meds hx
Psychiatrics, neurological symptoms

5 DEMENTIA SYNDROME
Alzheimer
Vascular
FTD
Lewy Body
Assocd with Parkinsons

DDX
Delirium tx underlying cause
B12 defy Wernicke (REVERSIBLE)
Thyroid dzs (REVERSIBLE)
Meds-related
SOL brain tumours
Pseudo-dementia (depressive disorders) ask about mood
Hypoglycaemia
Infections encephalitis (usually acute) - HSV
Alcohol-induced
Post-hypoxia syndrome
N P hydrocephalus

INVESTIGATIONS
MMSE (Mini-Mental State Examination)
o best, quick screening tool
o >26: N, 20-25: mild, 10-20: mod, <10: severe
MOCA (Montreal Cognitive Assessment) by OT
CAMCOG (Cambridge Cognition)
FAB (Frontal Assessment Battery)
ACE-R (Addenbrookes Cognitive Score) by OT
*flaws: cognitive decline may be masked among higher-educationed pts

Blood
B12, thyroid (reversible), folate (in alcoholics)
full blood screen: glucose, toxicology, FBC (WCC, anaemia), Fe studies (haemochromatosis), Ca + bone profile (hx of
prev malignancies myeloma), screen for neurosyphilis (if hx suggests)
U&E

Imaging
CT brain (1st), MRI outrile SOL, hydroceph, chronic vasc injury, atrophy even if N, do NOT rule out dementia (HX
IMPORTANT!), PET & SPECT (not routinely done in practice)

Others
In young: EEG, LP
brain biopsy rule out vasculitis
CASES
1. A 75 year old man presents to the Geriatric Medicine outpatient clinic as a referral from his general practitioner.
He has a two year history of gradual onset short term memory loss, forgetting names, losing his keys etc. He has
stopped engaging in his usual weekly card game with friends and his wife has taken over managing the finances at
home.

DDX
Alzheimers dementia
o Amnesia (semantic memory & immediate recall) happen first, episodic
o aphasia, agnosia, speech difficulty, visuospatial probs, behavioural disturbance + psychological (later on)

2. A 75 year old man presents to the Geriatric Medicine outpatient clinic as a referral from his general practitioner.
He also attends the diabetic clinic. He has a two year history of memory loss, which seemed to come on quite
quickly initially. He deteriorated significantly after a urinary tract infection last year and again after an admission
to hospital for a TIA. He has also had a number of falls over the last year.

DDX
Vascular
o Step-wise deterioration, assocd cdiac RF (high BP, smokes, PVD), personal fam hx, focal neurological symptoms, risk
assessment (falls-prominent gait issues), urinary incontinence early on , contributing factor (UTI in this case)
o Memory later on (x as prominent as Alzheimer)
o Exam: Afib, carotid bruits, focal neurological signs

3. A 75 year old man presents to the Geriatric Medicine outpatient clinic as a referral from his general practitioner.
He has a two year history of some mild memory impairment which seems to fluctuate over the course of the day. In
addition, he often becomes abruptly drowsy for short periods of time a few times a day. He reports sometimes
seeing people in the room who arent real. He also feels that his mobility has deteriorated and reports being stiff.

DDX
Lewi Body Dementia
a)Cognitive fluctuations
o Fluctuating attention: episodes of drowsiness, staring into space & obtunded (absence-type episodes), long naps
during day, disorganized speech
b)Visual hallucinations
o colors, shapes, animals, people auditory/tactile/touch with good insights that they are not real
c) Autonomic
o Parkinsonisms (TRAP) if given anti-psychotics, parkinsonism will worsen, tremor not as prominent as in
Parkinson
o Sleep difficulties
o Depression: co-morbid
o Poor regulations of body function: Low BP, pulse, sweating and digestive
MRI can see Lewy body :focal atrophy of the brain
Histology lewy body (eosinophilic intracytoplasmic nuclear inclusions due to aalph-synuclein conformation)

MX
Biological tailored to MMSE (level of severity), do not alter dzs course
ACHerase inhibitors- CId in pts with heart blocks, hx of ulcer dzs
NMDA antags
Anti-depressants- start at low dose, at later stage
RF
Vascular
Social
MDT social worker, carer supports, need for residential care, OT, physiotherapies
6.BONE HEALTH

DEFINE
Osteoporosis loss of bone marrow density & increased risk of falls
Osteopaenia decrease in bone density, not as severe as in osteoporosis but still at risk of falls
T-score comparisons of bone density in young populations
Z-score comparisons of bone density in an age-matched population
Fragility fracture falls risk at standing height that results in fracture (vertebral)

CLINICAL PRESENTATIONS
Fractures fragility fracture
Stoop posture: chronic compression/ wedge fractures which worsened over time crumbles down loss of height

TX
Pharmacological
Calcichew & Vit D: horrible taste, big, chalky diet Vit D tablets
Biphosphonates regurgitation risk (need to sit upright for half an hour after taking it), biphosphonates holiday
(should only be given for 5 yrs cause hyperthickened cortices causing lost flexibility snap and spiral fracture to
shaft of femur = biphosphonate farcture)
o CId in: gastric ulcers, eGFR<30, swallow disorders
o Compliance issues: alendronate -give once a wk, once a yr IV zalendronate injection. SE: flu-like symptoms for a
week afterwards
Parathyroid hormone stimulates new cell growth (risk of cancer!). excellent for vertebral fractures
Raloxifen (SERM) risk DV, PE
Strontium risk of MI
Testosterone replacement: for hypogonadism
Denosumab
Non-pharmacological MX
Weight-bearing exercises
Smoking cesssation

CASES
1. A 55 year old woman attends her GP. She is concerned about the possibility of osteoporosis. She has a history of
hypertension and takes amlodipine 5mg od. She fractured her ankle after tripping on the stairs eight years ago.
She is a smoker and she went through the menopause aged 50.
RF: female, post-menopausal, hx of fractures (not fragility site though), smokers
Other: increased age, thin (fat stores & produces estrogen)
DEXA Scan: reasonable to be done

2. A 78 year old man presents to the Emergency Department after a fall at home while under the influence of
alcohol. He has sustained a fracture of his left hip as a result.
RF: age, prev fall, alcohol (direct effect & malnutrition from alcohol)
Others: hypogonadism, hypopituitarism (prolactin production amenorrhea estrogen suppression), meds (steroids,
anti-epileptics, immunosuppressants - cyclosporin)
Ix: Blood FSH, LH, thyroid, parathyroid, screen for: Cushings dzs, IBD, Coeliac, CKD (affecting Vit D), malignancy
(leukaemia, lymphoma, myelofibrosis, multiple myeloma Serum protein electrophoresis)

3. An 82 year old woman attends the Geriatric Medicine clinic. She lives alone in a two storey house. Her daughter
reports she has become more unsteady on her feet in recent times and has had three falls in the last four months.
Her posture is slightly stooped but she does not complain of any back pain.
3 factors which may increased risk of falls
o prev hx of falls (biggest risk!), being female, housing environment, stooped posture, probably on multiple meds,
cognitive issues
*outrule: postural hypotension, cardiac arrhythmias (VT, fast Afib, SVT ( no hypotension, young)), bradyarrhythmias
(heart block- monitor with telemetry & holt monitors), hypersensitivity of carotid receptors
LEGS: arthritis, decreased musc mass, periphereal neurpathy + loss of proprioception
HEAD: cognitive decline (infarcts), intracranial issues, Parkinsons, meds mimicking Parkinsons (bipolar dzs)
3 interventions
o railings, walking aids,
DEXA Scan indicated
Friday, 11/9/2015
7.THYROID DISEASE
DEFINITION
Thyroid acropachy: clubbing painful finger & toe swellling due to periosteal reaction in bones of extremities
Pretibial myxedema: waxy discoloured induration of skin on shin spreading to distal areas of legs
Goitre: enlargement of thyroid gland swelling on neck
Exopthalmos: retractions of both lids & protrusion of eyes
Myxedema coma: complication of hypothyroidism (decompensated): life-threatening, hyperthermia, comatose
Thyrotoxic storm: complication of hyperthyroidism, life-threatening, high fever, fast irreg heart beat, diarrhea

CASES
1. A 55 year old woman presents to the Endocrine outpatients as a referral from her general practitioner. She
complains of lethargy, weight gain and hair loss.
Symptoms hypothyroidism:
Cold intolerance
Irregular period
Infertility
Constipation
Dry skin
Myalgia
Decreased concentration
Depression/ low mood

Signs of hypothyroid:
Bradykardia
Dry skin
Goitre
Oedema (periorbital oedema)
Carpel tunnel
Reduce ankle reflex
DDX
Hypothyroidism: Hashimotos, iodine defy, De Quervains thyroiditis (can cause hyperT as well)
Hypopituitarism
Meds: amiodarone, lithium, carbimazole
Pregnancy (child-bearing age)
Diabetes
Haemochromatosis
Viral illness
Anaemia
Depression

Risk factors:
Female
Genetics
Autoimmune: DM, coeliac, turners, trisomy 21
Drugs: amiodarone, carbimazole, lithium
Radiation
Hyperthyroidism after treatment
Pregnancy

Main cause:
Primary cause
Hashimoto- anti TPO
Thyroditis; px with hyper first then go hypo. ESR, Pet scan (no uptake)
Iodine def
Drugs
Pregnancy
Thyrotoxicosis can eventually
Thyrodectomy
Infiltrated: haemochromatosis (iron)

Subclinical 58%, hypo 2%


Tx or not: symptoms, artherosclerotic dz (high risk- low threshold to tx)
TSH > 10, normal T4: treat
TSH 5-10, if +ve Abx: treat
If they want to pregnant: treat

Secondary cause: pituitary

INVESTIGATION
TFTs: fT4 low, TSH high (if borderline (sick thyroid syndrome?) repeat in 6-8weeks)
AutoAb: TPO (Hashimotos), Anti thyroglobulin
FBC: anaemia (macrocytic)
Lipid profile: to screen for complications
US: if found nodular assymmmetric goitre in exam (adenoma) found nodules US-guided FNA biopsy

KEY CLINICAL FEATURES


Lethargy, weight gain, hair loss
Cold intolerance, menorrhagia, constipation, low appetite
Sign: bradycardia, dry thin skin, cold peripheries, goitre, puffy hands, reduced reflex, carpal tunnel syndrome, prox
musc weakness, ataxia, myxedema

MX
Levothyroxine 1.6mcg/kg/day
-elderly give 25mg, if in 50s give 50mg. check 4-6weeks.
Repeat TFTs every 6 weeks (normalized TSH)
Start at lower dose in elderly (risk of MI & angina)
If stable 6m-1year
Aim for normal TSH!!!!

Complications:
Myxedema coma: triggers by infection
2. A 55 year old woman presents to the Endocrine outpatients as a referral from her general practitioner. She
complains of neck swelling, hoarseness and intermittent palpitations. She also feels the appearance of her eyes
has changed in recent months

DDX
Hyperthyroidism - Graves dzs (60-80%)
Thyroid adenoma
Pituitary adenoma
Oropharyngeal ca
De Quervains thyroiditis
Post-partum thyroiditis
Struma ovarii
Meds: amiodarone

Causes:
Hyperthyroidism - Graves dzs (60-80%)
Multinodular goitre
Thryoiditis
Pregnancy
Drugs: Amiodarone, levothyroxine
Strumi ovari

Deranged TFTs: sick thyroid syndrome


INVESTIGATION
TFTs: low TSH, high fT4
AntiAB: TPO, anti TG, anti TSH receptor antibody (graves)
Radionuclide scans: diffusely increased uptake
-if low/ absent: ectopic production, thyroditis, struma ovarii, med-induced (amiodarone), iodide-rich food intake.
If nodular = hot , cold = malignant)
-pregnancy CI
Thyroid US FNA biopsy

KEY CLINICAL FEATURES


Sweating, weight loss depsite increased appetite, oligomenorrhea, insomnia, irritability, low mood, diarrhoea, heat
intolerance
Signs: Fine tremor, Afib, tachycardia, warm skin, palmar eryhthema, goitre, s3 murmur, gallop rhythm, pretibial
myxodema

Graves: exophthalmos, lid retraction, lid lag, opthalmoplegia, loss of colour vision, papilloedema, pretibial myxedema,
thyroid acropachy (formation of new bone)
Bilateral, symmetrical, non-pitting,
MX
Symptomatic
Intermittent palpitations & tremor: propanolol (b-blocker) not asthmatic!
Hyperthyroidism
Carbimazole teratogenic (1st line): blocks thyroid peroxidase (TPO) hormone production
***warn about agranulocytosis (sore throat, PUO, infections stop carbimazole and come to hosp)
PTU (if carbimazole not tolerated, or 1st trimester pregnancy): blocks peripheral conversion of T4 to T3
***agranulocytosis, jaundice (from permanent liver failure)

If meds not working


Radioiiodine: destroy thyroid tissue, indicated for Graves, toxic MNG, toxic adenoma (lat 2, tx with meds first then
radioiodine or surgery)
CI in pregnancy
SE: can go hypothyroid, Graves- eyes can get worse

If still symptomatic
Surgery
o Indication: symptoms of obstruction (Pembertons sign), meds failed, suspicious nodule, CI/ intolerant to meds
Post-surgery:
o Hoarseness (damage to RLN), hypoparathyroidism (tingling due to hypoCa)), bleeding (stridor, SOB, neck swelling)

Subclinical hyperthyroidism:
Rf for cardiac dz (IHD)
Symptomatic: afib
Post-menopausal: risk of osteoporosis
Progression to clinical hyperthyrodism
Pregnancy
3. A 55 year old woman has routine bloods performed by her GP. Thyroid function tests show TSH 0.05mU/L (0.5-
5.5mU/L) and T4 11pmol/L (9-22 pmol/L). Her usual medications include amlodipine 5mg,
calcium/cholecalciferol 500mg/400i.u. and alendronic acid 70mg

DDX
Symptomatic subclinical hyperthyroidism, goitre (5%)
o Cause: early Graves dzs, meds(lithium, levothyroxine), toxic multinodular goitre, thyroiditis, pregnancy,
hyperemesis gravidarum
Pituitary hypothyroidism (TSH will be lower)

INVESTIGATIONS
TFTs: repeat, take full hx & exam

Key clinical features


Asymptomatic
Calcium/ cholecalciferole & alendronate: osteoporosis
Amlodipine: CV dzs

MX
Tx if:
symptomatic
Afib
recent fractures (osteoporosis)
MNG

Thyroid toxic storm


Temperature: high, palpitation, sweating, vomiting, diarrhoea
Signs: confuse, multi organ failure

Treatment:
Iv fluid
Treat infection/ sepsis
Propanolol (oral- 60-80mg 4hourly)/ IV- 2-5mg per hour (make sure not asthmatic!!!!)
Anti emetics
PTU > carbimazole block conversion of t4 to t3 :NG- 200-300mg 4x a day
Lugol solution: give once 6 hours
Steroids
Digoxin- heart failure

8.DIABETES MELLITUS
DEFINE
DM: metabolic, hyperglycaemia. Type I (autoimmune). Type II (insulin resisatance & impaired insulin secretion)
Polydypsia: inceased thirst
Charcots joint: neuropathic arthropathy due to degeneration of weight-bearing jts with flattend natural arches
Acanthosis nigricans: brown hyperpiigmentation, common in neck, arm areas
Kussmauls breathing: laboured breathing assocd with met acidosis in DKA
Basal bolus regimen:?
Rubeosis Iridis: neovascularisation of iris
DKA: acute life-threatening, happens in Type I DM, state of hyperglycaemia, ketonuria/ ketonaemia, acidosis
Mononeuritis Multiplex: damage to at least two separate nerves areas (non-symmetrical, random, > symmetrical in
later stage as > nerve area affected)

DDX for hyperglycaemia (5Is)


Infarction in pancreas (pancreatitis)
Infection
Infant (Preganncy)
Institution (cocaine-induced or othe recreational drugs)
Insulin lack
2y causes of DM
CF, pancreatitis, haemochromatosis, Cushings dzs, acromegaly, steroids, pheamocrhomocytoma

CASES
1. A 26 year old man is brought to the emergency department after feeling unwell at home. He is previously well
and has no past medical history. On presentation he is found to be severely dehydrated and drowsy with a GCS
of 14/15. He reports some abdominal pain. BP is 98/50mmHg, HR 117/min, saturations 97% on room air,
temperature 37.7C, RR 28/min. Capillary blood glucose is checked and reads 29.4mmol/L
FURTHER QX
Resp, urinary symptoms
Infections: fever, night sweats, rigor
Precipitating factors: recent alcohol intake, recent stress, trauma or surgery, dehydration level

DDX
DKA: abd pain + hyperglycaemia, dehydrated, drowsy, ustable (hypotensive, tachycardic, tachypneic)

DIAGNOSTIC INVESTIGATION
Urine dipstick: ketones
ABG (VBG would be enough but ABG usually done because pt tachypnic): pH, LDH

ADDITIONAL INVESTIGATIONS
ECG: effects of insulin
Septic screen
LFTs, coag profile

KEY CLINICAL FEATURES


Symptoms: Decreased consciousness, lethargy
Signs: Decreased skin turgor, dehydration, ketotic breath

MX
SHORT TERM LONG TERM
IV fluids: 0.9% IV saline 1L STAT MDT: endocrinologists, diabetic nurses (HbA1c control
o 1L over 1hr, over 2hrs, over 4hrs, 6-8hrs & compliance), dietitician, podiaitrics, opthalmologists,
IV Insulin nephrologists, cardiologist, neurologists
o 6-10units of IV actrapid (short-acting)
o commence iV insulin 6U/hr Start Insulin regimen if first presentation
Capillary blood glucose If established, review meds (compliance, SE)
o every 2hrs
o once CBG<12mmol/L (N) after 2hrs, change to GKI
Repeat U&E after insulin infusion
o check U&E n ECG potassium
o if serum K < 5.5mmol/L, start K replacement
o monitor U&E, glucose, venous bicarb, ECG, urine output
LMWH prophylactic to prevent thromboembolic events
o Refer to local hospital mx
2. A 59 year old woman attends her GP for a check-up. She has a previous history of asthma, depression and high
cholesterol. She takes escitalopram 10mg daily and a salbutamol inhaler on a PRN basis. Routine blood tests
are performed. Fasting glucose is 9.6mmol/l. Full blood count is normal; urea is 5.1 with creatinine 150

FURTHER QX
Complications- eye (blurry vision,), renal, neuro (sesory deficits in glove & stockings distribution), MI, angina,

DDX
TIIDM

DIAGNOSTIC INVESTIGATION
Symptoms of hyperglycaemia
Plasma glucose> 7, OGTT: >11.1, HbA1c>48 (6.5%)

ADDITIONAL INVESTIGATIONS
Look for 2y causes of DM: Type I (Ab), urinary U: Cr ratio, eGFR, LFT, TFTs
Look for complications of DM: Type II

KEY CLINICAL FEATURES


Weight gain, polydipsia, polyuria, paraesthesia, ulcers in P areas of foot, hypothermia

MX
Conservative
lifestyle modifications for 3months: exercise
Meds
oral hypoglycaemics (metformin), Statins, Anti HTN,
Mx complications

Type I Type II
Metformin Insulin
CI in renal failure Basal (LA: Insulatard, Glargin, Levetemir)
Bolus (SA)
Sulfonylureas Pros: tight control
SE: hypoglycaemia, disulfiram-like Cons: 5-6injections/day: (invasive, lipodystrophy, risk of forgetting injections)
effects
Mixtures
Poor control

Pump
Tight control, hard to carry
SE: hypoglycaemias/ DKA if fail to work
Monday, 14/9/2015
9.COUGH
CASES
1. A 30 year old man presents with shortness of breath, cough, and wheezing that worsen in cold air. He has had
several such episodes in the past 4 months.
DDX
Chronic asthma: triggered symptoms (smoking, exercise, weather), atopy
COPD (Chronic bronchitis): if cough productive, A1AT defy. Less likely for this age & non-smoker
PF: if cough dry
GORD: chronic cough, could trigger & worsen asthma
Pneumonia: if acute
Meds: ACEi vagus nerve irritation, failure to inhibit bradykinin (increased)
Allergic rhinitis
TB: not cause no haemoptypsis, on exposure, night sweats do CXR (consolidation/ air cavitations at apices) +
Mantoux test (not reliable if ve, cud mean recently vaccinated, -ve useful), culture (6weeks to culture TB), Z&N stain
(quicker), isolate if actively produce sputum
*RLN palsy (any age, risk from recent surgery, hoarseness & stridor)

MX
Bronchodilator: B agonists (SABA PRN, LABA regularly)
ICS
Leukotriene antag: montelukast

2. A 56 year old woman presents with shortness of breath and productive cough that lasts for at least 3 months
each year in the past 2 years. She is a heavy smoker.
DDX
COPD (chronic bronchitis)
New onset asthma: age < likely
Bronchiectasis
ILD: IPF- occupation: coal mines, silica, exposure to asbestos fr buildings, farmer (pigeons fancier, hays), chronicity,
meds, PMH RA, inhalational heroin
Recurrent aspiration pneumonia in right lung base: Right main bronchus (> straight path). Why aspirate? Intubated (in
ICU or on tracheostomy), achalasia, oesophageal stricture, stroke pts whos on thickened diets
HF: not because no orthopnea & PND

3. A 58 year old man presents with 1 week of pleuritic chest pain, fever, chills, and cough with purulent yellow
sputum. He is a heavy smoker with COPD.
DDX
Bacterial pneumonia: Bacterial (purulent sputum), on PE: dullness to percussion , increase to vocal resonance, coarse
early inspiratory crepitations (infections) VS fibrosis (fine volcro-ey creps) VS fine in pulm oedema
Asthma
TB

INVESTIGATIONS
FBC: WCC high (if low: if already severe sepsis), cell differential (neutrophils high if bacterial, lymphocytosis if viral), pL
high (acute phase response- infection, bleeding, sepsis), Hg (low-anaemia, could be high-polycythaemia in chronic dzs)
CRP
Blood cultures
CXR: consolidation

TX
Ab: CAP (co-amoxiclav + clarithromycin, flucloxacilin or cephalosposrin (could also be allergic to ceph) if penicillin
allergic

4. A 55 year old man presents with increased dyspnoea and sputum production for the past 3 days. He has COPD
and stopped using his inhalers last week. He stopped smoking 2 days ago.
DDX
Acute exacerbation of COPD
o non-infective: identifiable triger
o infective: sputum production
Asthma
Bronchiectasis
Foreign body aspiration

INVESTIGATIONS
Peak flow meter

5. A 34 year old female nurse presents with worsening cough of 6 weeks duration accompanied by weight loss,
fatigue, night sweats, and fever. She has a history of contact with tuberculosis patients at work.
DDX
Pulmonary TB: worsening cough + systemic (wt loss + fatigue +night sweats + fever) + hx of contact with TB pts
Hodgkins lymphoma: B symptoms fever, night sweats, wt loss + age. Esp if has lymphadenopathy, young
Pulmonary carcinoma: wt loss, chronic
Exacerbation of COPD: < likely for age & non-smoker
Atypical pneumonia: if acute, esp if productive purulent cough?. Legionella & mycoplasma, if HIV (seroconversion
syndrome systemic symptoms)
Sarcoidosis: dry cough
ILD

MX
RIPE meds for 8 weeks, follow up for 16weeks while waiting for C&S results
LFTs (rifampicin cause LFT derangement), eyes (colour vision, Ischihara test optic neuritis caused by ethambutol)
work: stopped fr sputum production until resolved

6. A 50 year old male presents with a cough that is exacerbated by lying down at night and improved by propping
up on 3 pillows. He also reports exertional dyspnoea.
DDX
HF: PND, orthopnea, adding pillows
+ask: cardiac RFs (HTN, angina ,smoking, high chol, alcohol-dilated cdiomyopathy, fam hx0
10.WHEEZE
CASE
1.A 56y/o Caucasian man comes to the ED in an ambulance with worsening dyspnoea, fever, cough, and increased
purulent sputum production which is yellowish-brown in colour. He is accompanied by his sister, who informs you
that he has been increasingly short of breath and fatigued over the last 2days and has not been thinking clearly.
His sister also states that has had a cold for the past 5days, which he tried to mx with Paracetamol.

Past medical history Vitals


smoker for 30 years and has quit one year ago when he was Blood pressure - 130/84 (normotensive)
diagnosed with stage II (moderate) chronic obstructive Respiratory rate - 30/min (tachypneic)
pulmonary disease. Since being diagnosed, he has been Heart rate - 120/min (tachycardia)
taking salbutamol PRN and tiotropium bromide (Spiriva) Oxygen saturation - 85% (hypoxia)
daily. Temperature - 38.5C (pyrexial)
He has no other medical conditions, and no known allergies. He is using accessory muscles to breathe, has audible
expiratory wheezing and inspiratory crackles, and
diminished breath sounds in lower lobes upon auscultation
DDX
Acute infective exacerbation of COPD (no changes on CXR VS pneumonia)
o change in amount and colour of sputum, fever, worsening dyspnoea
Pneumonia: consolidation on CXR: clinical & radiological dx
Pulmonary abscess: foul smelling sputum, rigors, spiking temp, can dev septicaemia, haemoptysis
Lung cancer: can cause obstructive pneumonitis, wt loss, clubbing
**COPD: chronic irreversible progressive obstruction of airways characterised by chronic bronchitis & emphysema

INVESTIGATIONS
ABG: looking for acidosis, signs of resp failure
Peak flow meter: monitor reversibility after bronchodilator tx distinguish asthma VS COPD
FBC: Hg (polycythaemia), WCC (bacterial- neutrophils up), pL high (active infection)
CRP: raised
U&E: U&Cr: U elevated disproportionately compared to Cr in sepsis electrolyte imbalances
LFT: mets fr malignancy or legionella infection cause transamylitis + erythema multiforme
blood culture: is suspicious of TB, ask Z&N stain for AFB n TB culture
ECG: sinus tachycardia VS irreg (AFib)
CXR: consolidation, opacity (mass)

ABG Interpretation
pH 7.20 (acidic)
paO2 6.8 (hypoxia) cause confusion, GCS reducing, cyanosis
paCO2 8.9 (hypercapnia) CO2 retention (asterixis, palmar erythema, dilated vessels, bounding pulse, chemosis, confused
HCO3 (high)
*if both CO2 & HCO3 deranged, refer to one most deviated

Respiratory acidosis with metabolic compensation/ acidotic hypercapnic resp failure


Respiratory failure Type II (CO2 N or low (hyperventilation) in Type I RF)
Use of accessory muscles, tripod sitting position, tachypnic, hypercapnic (confused or even comatose!)

TX
Admit
2 large bore IV access
Acute symptomatic (ABCDE) After stabilisation
Nebulised controlled O2 (aim 88-92% sats 4L/min) + salbutamol Non-invasive ventilation
21% = in normal room air. 1L = 24%, every L increase 4% up to 10L biPAP (bi-level (delivered at
1st with nasal prong inspiratory & expiratory
if still needed: facemask (non-rebreather: takes away whats exhaled (has one-way valve level): to help blow out CO2)
with perforations on mask) or venturi mask) used in Type II RF (also caused
GCS by myasthenia gravis, GB,
Nebulisers poliomyelitis)
salbutamol (Ventolin) (SE: tremor, hypokalaemia- used in hyperkalaemic renal failure for CPAP(continuous positive
short term relief, tachycardia (put on this pt on cardiac monitor since he already has sinus airway pressure): used in Type
tachycardia) I RF-PE, PF, pneumonia,
ipratropium bromide (Combivent-2.5mg salb +IB or 5mg if need higher dose) (SAMA: SE- pulmonary edema
dry mouth, dry eyes, constipation in long term & with high dose)
IV steroids
hydrocortisone 100-200mg QDS after responded prednisolone 40mg
Antibiotics
amoxycilin + clarithromycin (CAP, atypical pneumonia-legionella, mycoplasma (test with
urinary legionella Ag, mycoplasma serology))

Friday, 18/9/2015
12. MURMUR
Systolic
TYPES AS (LV systemic) MR (LA LV)
SYMPTOMS SAD: Syncope, Angina or Dyspnoea SOB, fatigue & weakness
Outlet obstruction hypoperfused, especially in Palpitations (blood backflow into LA in turbulent
conditions requiring high O2 requirement way SA node hyperactivity (AFib)
Ex: when exercising increase preload, increase
EF (flawed in AS)
APEX BEAT Prominent non-displaced apex beat: heaving (P- Extremely displaced apex beat
loaded due to stenosis: calcification & bicuspid Hyperkinetic/ forceful (due to volume overload in
valve) ventricles from leaking & backflow)
PULSE Low vol, slow rising, parvus et tardus, delayed Jerky (some volumes ejected, some backflow)
CHARACTE upstroke (P building behind a stenosis blood
R pushed against a significant P, not as successful)
Narrow pulse P: small difference between systolic
& diastolic BP (stenosis is blocking decent
contraction/ systolic systolic BP decreased)
BP & JVP a wave: in AS, only in late stage Normal
oP back into LV LVH LV dilatation
(decompensation) apex beat change:
displaced LVF (PND, orthopnea, dyspnoea)
RVF (P build up, goes backwards to RV)
RVH (prominent a wave in JVP!))
HEART HS1 (closure of mitral & tricuspid) & HS 2 HS1 affected, soft HS1
SOUNDS (closure of aortic-1st & pulm-2nd) are audible In acute MR (papillary muscle rupture)
HS2 affected: soft (due to lots of Ca deposition In chronic MR (LV dilatation: cause mitral valve
causing the stenosis) but in congenital (slam-hard moves apart functional MR), MVP, RF, IE
sound) Added sounds: S3
A2-P2 splitting reduced (due to A2 closes slower)
Added sounds: S4
MURMUR Ejection systolic (S1-pause-murmur-pause-A2 P2: Pansystolic
gap bet S1 &S2)

TYPES VSD (LV RV) HOCM


SYMPTOMS 1/4 in neonates, 5% in adults depending on Similar to AS
& SIGNS geography Asymptomatic
RVF symptoms: dyspnea, oedema, pulm HTN Angina
(haemoptysis) Palpitations
oL: high P systems shunts to R o conduction disturbance due to hypertrophy of
Recurrent resp tract infections + raised JVP + muscles distortion of pathways
hepatic congestion (hepatomegaly, may lead to Dyspnea, arrhythmias, sudden death
chirrosis rarely) Failure of contraction & filling: outlet problem
L parasternal heave (hypoperfusion, hypertrophy)
oRV & RA hypertrophy
APEX BEAT Normal Usually non-displaced
If got to stage of LV dilatation displaced (in 50%
cases)
PULSE Normal Bifid: 2 impulses
CHARACTE
R
BP & JVP Normal Low (low output) later heart tries to
compensate systolic HTN
HEART S1 N S2 (aortic valve affected in systole)
SOUNDS Reduced splitting or A2 & P2 occuring at same Added sounds: S4
time
owhen pulm HTN occurs: loud P2 (big P behind
pulmonary valve)
MURMUR Pansystolic Depends on size
o pansystolic
o can be mid-systolic if < LV outlet obstruction

OTHER: PS, TR
Diastolic: written paper (theories)
TYPES AR (Aorta LV) MS (LA LV)
SYMPTOMS CCF Dyspnea, palpitation (from AFib)
Pulm HTN if chronic (haemoptysis & hoarseness) If chronic: pulm HTN RHF (oedema etc)
Mitral facies/ malar flush (congested vessels, pulm
capillary breaks)
APEX BEAT Displaced in long term Non-displaced (LV salvaged! =))
Tapping (high P system at level of mitral valve)
PULSE Pulsations all around body Normal
CHARACTE oradial: collapsing/ Waterhammer big effort
R to push blood out but then fall backwards:
cant get out through leaky valve
oCorrigans pulse in carotid
oQuinckes sign
BP & JVP Wide pulse P (big difference in systolic & diastolic Normal
BP: ex 100/40)
HEART S2: soft (leaky, not ejecting properly) S1 affected
SOUNDS No added sounds oRHF main cause, carcinoid syndrome, lupus
osoft (usually) or loud (depends on aetiology)
Prolonged and louder S2
Loud P2 (P in P component higher than N = no
splitting)
MURMUR Early diastolic Rumbling low-pitch mid-diastolic

OTHERS: TR, PS

TX
Valve replacement for all valves disease except in HOCM
TYPES TREATMENT
AS Low dose ACE-I & diuretics
Valvuloplasty
***Anti HTN meds (nitrates & B-blocker): will decrease preloads contraindicated!
MR Symptomatic control of CCF: anti HTN meds can be used
Mx of AFib: anticoagulation (rate control +/- rhythm control)
VSD Depends on size
osmall hole = louder, large hole = softer, therefore soft murmur should worry more!
Mx of RHF: salt restriction, volume reduction
HOCM Transplant curative
Increase volume: give fluids, phenylephrine (selective alpha-1 adrenergic receptor agonist, a
decongestant and vasoconstrictor)
Mx CCF
If gets recurrent arrhythmias: CRT (ICD), B-blocker
AR Mx CCF
MS Mx symptoms of RHF
Mx AFib

Additional informations
JVP = RAP
oa wave increased: RVH, pulmonary HTN, MS, AS (if end-stage)
AS & MR: has significantly reduced as < RF nowadays
VSD, HOCM and sometimes in MR: murmur heard pan-precordium
PS & TS seen in early life

INVESTIGATION
ECG
oFeatures of RHF: RVH, RBBB, R axis deviation
oFeatures of LHF: LVH, LBBB, L axis deviation
oFeatures of arrhythmias: AFib & P. Mitrale (AbN in mitral valve bifid) in MS & MR
ECHO: EF, pressure across valves, end-diastolic volumes
CASES
1. A 50 year old male with a known heart murmur presents with complaints of substernal chest
pain (angina), which increases with exertion, and shortness of breath which is starting to limit
his lifestyle. He has no risk factors for coronary artery disease.
On physical examination you find the following:
Delayed carotid upstroke (slow rising)
A sustained apical pulse
Prominent A wave in the neck: pulm HTN/ PS/ TS/ late stage AS
PMI is sustained but not displaced laterally: LVH usually
**PMI: point of maximal impulse/ apical impulse (sustained apex beat in all/ almost all systolic phase)

2. A 45 year old male with a history of rheumatic fever presents with progressive shortness of
breath and dyspnoea on exertion and is progressively getting worse. He has also developed
intermittent complaints of palpitations.
The following are noted on examination:
Increased respiratory rate
Normal PMI
RV lift
Increased JVP
Crackles on lung exam

3. A 52 year old female presents with complaints of slowly progressive dyspnoea on exertion
and an uncomfortable awareness of pulsations in the neck and chest.
On examination the following are noted:
Abnormal brisk pulses
Wide pulse pressures (AR)
Quinckes pulse: redness & pallor seen under fingernails (AR)
Head bobbing: respiratory distress
Pistol shot sounds (Traubes sign): a loud, cracking sound heard by the stethoscope over an artery in
which there is distension followed by an abrupt collapse, as classically occurs in large arteries
(femoral artery)(AR)

4. A 75 year old male present to the emergency room with complaints of severe chest tightness
(10/10) and acutely short of breath. He has PND and orthopnea. He is hypotensive, tachycardic
and in respiratory distress. His ECG reveals an inferior and posterior wall myocardial
infarction.
On examination:
Vital signs are unstable
Crackles are noted bilaterally
PMI is still relatively normal
5.A 22 year old male presents for a routine physical exam. He was referred to cardiology
because of a murmur and wanted clearance to play sports. He has a family history of sudden
cardiac death.
On cardiac examination PMI is markedly sustained with a palpable a wave.

Monday, 21/9/2015
13.Dyspnoea

CASES
1. A 65 year old male presents with worsening cough for the past 6 months accompanied by
haemoptysis, dyspnoea, weakness, and weight loss. He is a heavy smoker.
2. A 65-year-old female smoker presents with worsening cough, dyspnoea, haemoptysis and 6 kg weight loss
over the last 6 months. On examination, she has a low body mass index, with oxygen saturations of 93% on
room air and reduced breath sounds and crackles in her right lung.

DDX
Lung carcinoma: chronic, haemoptysis, wt loss, heavy smoker
TB: +fever +night sweats +travel hx +immunosuppression +palpable LNpathy
IPF: -haemoptysis
Bronchiesctasis:
Lung abscess: +systemically unwell
Vascultis (Wegeners): +haematuria
COPD: +haemoptysis in aggressive COPD
Pneumonia: If severe
CCF: +fluid retention
AVM (arteriovenous malformation)
Atypical infection (aspergillosis)

WORK-UP
ABG: hypoxia, low chance of hypercapnia (look for signs of CO2 retention: asterixis, chemosis, bounding
pulse, dilated veins, drowsiness, confusion, agitated, restlessness)
FBC: neutrophils high, Hg (if haemoptysis- blood loss)
CRP, ESR
U&E: volume depletion, high urea (renal impairment)
LFTs: malignancy (liver mets)
Inflammatory markers: c-ANCA
Urinalysis: haematuria
Microscopy urine: look for cast

CXR: mass, areas of opacification (malignancy, TB, vasculitis pulmonary haemorrhage), abscess
If mass present on CXR:
o+CT thorax with contrast
o+bronchoscopy
PET scan- flurodeoxyglucose cancer will take up more as metabolic is higher)
Biopsy- endobronchial u/s (peripheral lung cancer)
Bone scan (pain at back)
Ecog perfomance status

3. A 35 year old male presents with shortness of breath and cough. He has had unprotected sex
with multiple sexual partners and was recently exposed to a patient with active tuberculosis.
4. A 35-year-old male non-smoker presents to his GP with a five-day history of worsening dyspnoea and a dry
cough. He has a history of previous intravenous heroin use and has been on a methadone programme for the
last three months. His oxygen saturations are noted to be 89% on room air. His housemate is currently on
treatment for tuberculosis.

DDX
TB
oif coughing & producing smear 3 early morning sputum sample do TB culture and check for acid fast
bacilli will only do routine bacterial infection
o***specifically ask f ZN stain: looking for AFB + TB culture (pt with ZN stain negative can still grow TB on
culture!)
Pneumonia
oP.carinii/ pneumosytic
Immunosuppressed: HIV (IVDU), inherited syndromes, transplants
Non productive cough, hypoxia, atypical CXR findings- diffuse & symmetrical opacification
CXR:
CT: ground glass
Gene expert:
PCR brachopulmonary lavage

Aspergillosis
Lymphoma

Management
-FBC: lymphopenia, TB (wcc increase)
-interferon gamma

INVESTIGATIONS
CXR: cavitating lesion
o***If not producing sputum give hypertonic saline (SE: infection risk)
So better do bronchoscopy
Or BAL: flushing down N saline, traps cells ,bacs & debris sent to lab (gain sample not just in bronchus,
but also further down in alveoli (alveoli macrophages indicative of alveoli sample) distally)
***how to look for subcarinal LN?? biopsy gained by endobronchial US (EBUS) usually in right main
bronchus (anatomy: straighter > common to get infection)

P.carinii dx: CXR, silver stain, PCR. TX: co-trimoxazole

4. A 56 year old female presents with a one week history of worsening dyspnoea. She has a past medical
history of diabetes mellitus, hypertension, hypercholesterolemia, obesity, and seropositive rheumatoid
arthritis (10 years). Current medications are methotrexate, folic acid, ibuprofen and etanercept. Her
mother had type 2 diabetes and her younger brother is an asthmatic. She reports tobacco use of 20 pack
years and no recent travel history.
5. A 56-year-old male non-smoker complains of a 3-month history of worsening dyspnoea. He has a history of severe
rheumatoid arthritis and is on methotrexate, folic acid and etanercept. On examination, he has swan-neck deformities
of his fingers, oxygen saturations of 94% on room air and crackles in his lower lung zones.

DDX
ILD: RA, methotrexate (cause pneumonitis not ILD)
o give co-trimoxazole prophylaxis
o clubing, cough, crackles
o aetiology: drugs
amiodarone: had thyroid dzs, slate grey skin tone, liver (deranged LFts), thyroid (deranged TFTs)
etanercept: TNF-alpha increase susceptibility to TB infection or reactivation of latent TB

ILD INVESTIGATIONS
HRCT: honeycombing, increased interstitial markings
CT with IV contrast: distinguish between LN or blood vessels
PFTs: restrictive FVC low, FEV1 N or low, ratio usually high or N, low residual lung volumes, Dlco
decreased, Type I RF can get pulm HTN
DLCO
Hypersensitivity pneumonitis: abestos,

Upper lobe fibrosis: Ank.spondylitis, sarcoidosis, silicosis, eosinophylic pneumonia,


4. A 40 y/o woman presents with SOB and dsypnoea on exertion for 1 day, sore throat, dry cough, fever,
chills for 7 days, myalgia and fatigue for 2 days. She denies orthopnoea or PND, has no sputum or chest
pain, no leg trauma or swelling, no travel or immobilization, no history of asthma, COPD, heart failure.
She is a non smoker, non drinker and takes no regular medications. She appears fatigued and sleepy. RR
32/min, pulse 120/min regular, BP 124/76 mmHg, T 37C, O2 saturations 88% on room air.
On examination:
Respiratory decreased breath sounds bilaterally, no wheezes, rhonchi or rales
Cardiovascular JVP not elevated, no gallop, rub, or murmur
Abdomen soft, non tender, no mass
Extremities no oedema or rash, no calf tenderness
Neurological GCS 15/15, PEARL (pupil equal & reactive to light), moving all extremities
Results:
WBC 12.5 (high)
Platelets 125 (low)
LFT and Renal are normal
ABG on RA
opH 7.26 (acidosis), pCO2 8 kPa (hypercapnia), pO2 8 kPa (hypoxia)
Type II RF, aetiology:
COPD: emphysema (alveolar destruction capillary damage impair O2 perfusion)
Resp muscle weakness (GB: recent infection, ascending musc weakness, areflexia, spirometry:
FEV1, FVC low)
Myasthenia gravis (Acherase Ab: Tensilons test, ptosis, fatigability of muscles)
Duchennes, MND
CXR - within normal limits

DDX
Viral infection- influenza virus

5. A 32 year old man presents to the ED with shortness of breath for 1 day associated with fatigue for the
last 3 days. He denies chest pain, has no fever, chills or sweats, no cough, sore throat or coryza, no travel,
trauma or leg pain, no history of asthma, COPD, heart failure. He is a non smoker, takes no alcohol and
is on no regular medications. Vitals - RR 30/min, pulse 125/min, BP 102/64mmHg, temperature 37C,
O2 saturations 99% RA. He is wide awake but has laboured breathing.
On examination:
Respiratory chest clear, good breath sounds bilaterally
Cardiovascular JVP not elevated, grade 3 ejection systolic murmur, no gallop or rub
Abdomen soft, nontender, no mass
Extremities no edema
Neurologically Alert, Oriented x 3. Moving all limbs.
Results
WBC 15.5 (high) ABG on room air
Platelets 325 opH 7.26: acidosis
Electrolytes: opCO2 3.0 kPa: hypocapnic
Na+ 128 hyperventilate: compensating for metabolic acidosis or panic attack
Cl 92 opO2 11 kPa
HCO2 48 (low) METABOLIC ACIDOSIS
K+ 4.7
ECG: sinus tachycardia
Urea 16 CXR within normal limits
Creatinine 3.6

DDX
***Anion gap mneumonic: MUDPILES
Methanol
Uraemia
DKA/ AKA
Paraldehyde (CNS depressants)/ Phenformin (biguanide)
Iron/ INH
Lactic acidosis
Ethylene glycol poisoning (anti-freeze)
Salicylates

6. A 37 year old man c/o SOB for 2 hours, with associated left pleuritic chest pain, weakness and
lightheadedness. He denies trauma, travel or leg pain. He has nocough, coryza or sore throat, no fever, chills
or sweats. He has no history of COPD, heart failure or asthma.Smokes marijuana regularly, but no cigarettes,
other drugs, alcohol or regular medications. Vitals RR 36/min, pulse 132/min, BP 92/60mmHg,
temperature 37C, O2 saturations 90% on room air.

7. A 45 year old male is referred to the ED with gradually increasing dsypnea on exertion for 6 days. He also
complains of diffuse pleuritic chest pain for 4 days with associated non productive cough. He denies sore throat,
coryza,fever, chills or sweats. There is no history of trauma, leg pain or long haul travel. He has no history of
asthma, COPD, heart failure or otherwise. He is a non-smoker, non-drinker and takes no regular
medications. Vitals -RR 26/min, pulse 120/min, BP 160/92mmHg,T 38C, O2 saturations 95% on room air.
On examination:
Respiratory nontender, diffuse mild end expiratory wheezing R>L
Cardiovascular JVP note elevated, grade 2/6 ejection systolic murmur at the left upper sternal border
Abdomen soft, nontender, no mass
Extremities 1+ bilateral pedal edema
Neurological Alert and Orientated x 3, Moving all limbs, PEARL
Results:
WBC 11.2
platelets 315
Renal and LFT normal
ABG on room air - pH - 7.42kPa, pCO2 3.2,pO2 - 9
ECG sinus tachycardia
CXR normal
8. A 65 year old man is referred to the ED with a 1 day history of shortness of breath. He has no chest pain,
cough, or coryza, no trauma, leg pain or swelling, no fever, chills, or sweats. He has a past history of COPD and
congestive heart failure.Medications furosemide, GTN, digoxin, KCl, albuterol, ipratropium, prednisolone. He is
a smoker with a 120 pack year history, drinks one litre of whiskey a day. He denies any illicit drug use.Vitals
- RR 16/min, pulse 132/min, BP 194/112mmHg,T 36C, O2 saturations 75% on 100% O2 via a non-rebreathing
mask.

14.PALPITATIONS
CASES
1. A 50y/o man is referred to the ED by his GP. He awoke in the early hours of this morning with an
uncomfortable awareness of a fast heartbeat; the sensation has continued since then. He complains that
his chest feels tight and he is slightly SOB and he felt light-headed while walking to the ED, but did not
faint. He had a number of similar episodes in the past, usually on waking and settling within about 20-
30 min. He has had no blackouts. He is increasingly breathless on exertion (last 2-3months, particularly
when walking briskly uphill). slightly breathless when lying flat, eased by sleeping with three pillows.

Skipped, raising, jumping out of chest

PMHx Hypertension and mild asthma.


Smoker 20 pack years
Alcohol 30 units/week Episode of binge drinking last night.

Vitals On Examination
Alert and talking, appears General
pale and slightly sweaty, Bilateral symmetrical ankle oedema
with cool extremities BMI 29kg/m2
Pulse - 140/min, irregular JVP not raised
BP - 130/70mmHg Chest
RR - 20/min Heart sounds are normal.
Sats - 94% (RA) Bibasal inspiratory crepitations and quiet expiratory wheeze throughout
both lung fields.

Telemetry - irregular tachycardia with a rate between 150 and 170 beats/min.
No P waves are visible and the QRS complexes all appear similar in
morphology
DDX
Afib: palpitation, hx of alcohol as precipitants, ECG findings (tachycardia + irreg rhythm (no p waves))
SVT: specific rhythm regular, very fast, faster than AFib (120-180bpm)
Aflutter: saw tooth

Aetiology of AFib AFib Types


Cardiac: IHD (cause conduction disease), structural heart New onset
disease (cardiomyopathy, atrial myxoma (rare!)) 2y to HTN Paroxysmal
Pulm: PE, pneumonia, carcinoma compressing on cardiac Persistent (>7days)
wall, pericarditis Permanent
Metabolic: thyrotoxicosis (must screen for TFTs, in acute AFib), Lone (no structural heart disease)
hypokalaemia, uraemia 2y to renal impairment
Toxic: Binge alcohol drinking, caffeine, cocaine
Iatrogenic: post-op
Sepsis
Thyroid
Anaemia
Psych: anxiety, depression, somatization, stress
SVT:
A fib
A flutter
AVN RT (75% young female)
AV RT
VT:
V. fib
V tach
Electrical:
WPW
LGL
Channels:
Pregabalin (Na+ channel)

Causes Afib
Cardiac

Correctable precipitation factors


IV fluids: dehydration
Alcohol withdrawal: Chlorzodiapoxide + Pabrinex (prevent Wernickes encephalopathy)

Investigations
Stable or not? ABC
oexclude MI with ECG (new onset of BBB, AV conduction delay (heart blocks), T wave inversion) & cardiac
markers (troponin & CK-MB)
oECG: a fib, increase QRS, Qwaves, LVH
oexclude acute PE with ECG (R heart strain)

FBC: Hg (anaemia: precipitates palpitations & HF), WCC (infection may precipitate AFib)
TFTs: thyrotoxicosis
U&E: hypokalaemia, dehydration
BNP
Telemetry: assess heart rate

CXR: heart failure signs (ABCDE), bibasal creps (bilat: HF, unilateral: consolidation or mass)
ECHO- EF, hypertrophy, valve-MS,MR, RMWA-regional motion of wall activity, thrombi- LAA, left atrium size-
normal < 4cm, >6cm abnormal. (take time, later)
***Bilat pleural effusions: chest drain not helping cause the fluid will keep coming out
MX
IMMEDIATE MX THERAPEUTIC
O2 Aim: to convert them to sinus rhythm not to
o95-96%, 90%if prev lung disease decrease rate
Diuretics Depends on:
oIV Furosemide 20mg, 40mg , 80mg (pulmonary Who is > likely to convert to sinus rhythm following
oedema + unwell = 80), orally if good condition AFib
oBendrofluorothiazide can be given orally only oacute onset, 1st presentation
If hypokalaemic: +K+ oyounger pts: chances of prev structural heart dzs
Bblockers low low chance of conduction prob
oCId in asthmatics bronchospasm
oIf not asthmatics Rate control: as above
IV metaprolol 2.5-5mg, <100/<80 BP CId Rhythm control:
do not give too high hypotensive Chemical
emergency! Require cardioversion Amiodarone: in acute setting, anticoag first
sync cardioversion if with pulse (capture Flecainide, sotalol, propafenone
existing pulse & resync it) Electrical
unsync if pulseless (unorganized shock Cardioversion (synchronised):
rhythm) o<48hrs (acute)
Digoxin 250-500mcg (loading dose) effective! TOE (exclude left atrial thrombus if
ohypokalaemia + renal failure predispose to present indicate chronic AFib, x happy to
digoxin toxicity keep the level low! give shock)
under sedation, give electrical sync
If still not resolving: cardioversion 300joules check later if
oCa channel blocker (used in routine outpt still on sinus rhythm (discharge with
settings) but not acutely anticoagulation) or x
o>48hrs (chronic)
oif convinced has had palpitations before:
anticoagulate for 4-6weeks first, then only do
TOE as above

RULES OF ANTICOAGULATION
Unless do not have evidence of chronicity (have to
anticoagulation, cannot shock will cause
stroke!!: large, MCA)
Acute, no major surgeries

Other tx: catheter ablation

2. A 59 year old man with a background history of hypertension, coronary artery disease and coronary
artery bypass graft surgery two years earlier, complicated by post-operative atrial fibrillation, presents
now with palpitations and pre-syncope. (see attached file for ECG)
DX
Pre-syncope secondary to AFlutter
o atria beating at max rate 300bpm ventricles can keep up at max 150-200 bpm (how many p waves
firing per Q waves)

DDX
Aflutter
AFib
SVT
Re-enterant tachycardia

INVESTIGATIONS
As in AFib

TX OPTIONS
Catheter ablation very succesful in Aflutter
**complications quite rare as compared to AFib
3. A 22 year old student is referred to the cardiology outpatients for recurrent, intermittent
palpitations. She has no past medical history otherwise. She remembers having similar episodes lasting
not more than 10 min since she was 17 years old. She tells you that she could terminate some of these
episodes by holding her breath. There appear to be no known triggers. There have been 2 episodes of
dizziness associated with palpitations during one of which she blacked out. The routine blood tests are
reported normal.
ECG
Narrow complex tachycardia
150-160
DDX: SVT regular

TX
Vagal maneuver: blow into syringe, < successful
Carotid massage
IV Adenosine flush (place fat pad under arm): very fast, stopping heart beat (awful feeling!) cause flat-
lined ECG
**in asthmatics: CId bronchospasm
Avoid precipitatnts: alcohol, volumd overload (pregnant)
Catheter ablation if recurrent

Friday, 25/9/2015
15&16.CHEST PAIN [Dr. Jagtaps version]

CASES
1. A 50 year old male with a past medical history of gastroesophageal reflux and tobacco use presents to
the ED complaining of left-sided, sharp chest pain that radiates to his back and down his left arm. He
also complains of shortness of breath, nausea but no vomiting. The pain started shortly after he ate
lunch at work when he was moving some boxes around and lasted a few minutes before it improving on
sitting down. Currently he has minimal pain in his left chest only
Physical Exam
Vitals: BP 165/100, HR 95, RR 20, temp normal, Sat 100% on room air
General Looks comfortable
Neck JVP @ 6cm
Chest Clear to auscultation bilaterally
CV Rate Rhythm Regular, mild II/VI Ejection Systolic Murmur left sternal border, no radiation
Abdo soft, obese, nontender
Extremities trace lower extremity oedema, good pulses in all extremities. Equal BPs in both arms

DDX
Exclude these life threatening conditions first! Other
ACS Atypical MI
PE Angina: pain relieved with rest
Aortic dissection Oesophagitis: past hx of GORD, sharp pain, precipitated
Pneumothorax with eating
Pericardial tamponade: +coagulopathy Oesophageal spasm
Aortic stenosis: after work-up
FURTHER QXS
SOB: duration
Syncope
Symptoms & RF of DVT
RF of MI: DM (atypical symptoms due to neuropathy), HTN, hyperlipidaemia, etc
Hx of angina
Fam hx of CVD or respiratory disease
Chest pain
ocardiac: >1/2 hour
Diaphoresis: > likely to have organic causes
PE: decreased breath sounds
oD-dimers increase in any events causing fibrinolysis (low predictive value)
Aortic dissection: radiation to back, CXR: widened mediastinum
oif in ascending aorta stroke

CARDIAC RF HELPFUL?
Yes, part of tx is to modify RF
osmoking, HTN, physical inactivity

DXIAGNOSTIC VALUE OF A PATIENTS RESPONSE TO TX


GTN- no significant value could also relieve oesophageal-caused chest pain

CAN A SINGLE VE TROPONIN EXCLUDE ACS?


Do serial tests: CK-MB rises first followed by troponin
LDH rises quickly & drops quickly
OTHER TESTS
CK-MB
LDH: cellular death, ischaemia
ECG:
o ST (STEMI thrombolysis or PCI )
o NSTEMI (anticoags (LMWH): 1mg/kg in cardiac, 1.5mg/kg in other clots (pulmonary))
o Pathological Q
2.A 52 year old morbidly obese male presents to the medical assessment unit with complaining of chest
pain that worsens when he takes a deep breath in. He is rather immobile and only walks around his
house with a frame. Past medical history includes: Diabetes Mellitus (non-smoker, no hypertension, no
coronary artery disease)
Physical Exam
Vital signs: BP 140/90, HR 105, RR 25, T afebrile, Sat 95% on RA
General Obese male, talks in short sentences
Neck Unable to see JVP, no bruits
Chest Very distant breath sounds but clear
CVS very distant S1/S2, no other audible sounds
Abdo Obese, non-tender
Extremities Good pulses in all extremities, bilateral lower extremity oedema symmetrical, below knee,
non-pitting

DDX
DVT/PE
Atypical MI: DM
Aortic dissection
Lung carcinoma: if smoking, haemoptysis, weight loss
Costochondritis

OTHER QXS
PE: SOB, chest pain, tachypnoea

CARDIAC RF HELPFUL?
Yes, DM-increase likelihood of coagulation

DIAGNOSTIC VALUE OF A PATIENTS RESPONSE TO TX


Wells score
oPE >likely than other dx >3times
oIf negative: does not rule out PE but need to investigate further with CTPA

CAN A CT ANGIO, D-DIMER, OR V/Q SCAN RULE OUT PE?


CTPA
oMay miss subsegmental PE at distant or end capillaries) if not available V/Q scan: show perfusion
mismatch
ECG
oUsually no changes, right heart strain (inverted T waves in V1-4) + sinus tachycardia + right axis changes
+ acute MI pattern (if PE large)
CXR
oHamptons hump: wedge-shaped opacity indicating infarction in lower base of lungs
oWestermark sign: L hemidiaphragm elevation + focal oligaemia
oFleishner sign: enlarged pulmonary artery
oPleural effusion

3. A 60 year old male with a background of hypertension presents with sharp/stabbing chest pain of
9/10 severity radiating to his back. He has a 20 pack smoking history but otherwise no cardiac risk
factors (no hyperlipidaemia, no diabetes, no family history)
Physical Exam
Vitals BP 200/120, pulse 135/min, RR - 30/min, afebrile
General Elderly male, obviously in pain
Neck JVP at 6 cm
Chest Clear to auscultation bilaterally
Cardiovascular Tachycardia with a regular rhythm, no murmurs, gallops or rubs.
Abdo thin, soft, non-tender
Extremities decreased left femoral pulse

DDX
Aortic dissection: sharp stabbing pain radiating to the back, smoking, HTN, unstable +syncope (10%)
MI: symptoms + smoking, HTN
Malignant HTN: BP (>200/130) if has retinal haemorrhage, headache, decreased L femoral pulse

FURTHER QXS
Nature: continuous but intermittent in severity

CARDIAC RF HELPFUL?
PMH of cardiac dzs, Marfan or Ehler Danlos syndrome

DIAGNOSTIC VALUE OF A PATIENTS RESPONSE TO TX


No value

WHAT TESTS/ LABS MAY U WANT TO ORDER?


CXR
owidened mediastinum
oabN aortic knuckle (80%)
owidened parasternal shadow
opleural effusion mostly on L side
otracheal shift
ocalcification
odepression of L main stem of bronchus downwards
GOLD STANDARD: MRI- shows 3D image of aorta (layers) if not available CT
TOE

Classification: Stanford

MX
Goal: reduce BP
Surgery
oIndication
haemodynamically unstable
occlusion of major artery (reanal, carotid, ililac)
aortic leaking or rupture
uncontrollable HTN
Type A in Stanford classification (ascending aorta dissections)
Meds: Anti-HTN (Bblocker IV labetalol, metaprolol)

4. A 26 year old, tall thin male presents with chest pain and shortness of breath starting suddenly this
afternoon while playing basketball. He does not smoke, drink or use alcohol. He denies hypertension,
diabetes, high cholesterol or a family history of heart disease.
Physical Exam
Vitals -BP - 110/70 mmHg, pulse 120/min, RR - 30, Afebrile
General Physically fit male, talks in short sentences
Neck JVP at 5 cm
Chest Decreased breath sounds on the right, no crackles or wheezes
Cardiovascular Tachycardia, Heart sounds - normal
Abdo thin, soft, non-tender
Extremities no oedema, good pulses in all extremities

DDX
Pneumothorax
otall thin male, age, active, acute onset dyspnoea, no significant RF of cardiac disease
odue to pleural bleb rupture
Acute asthma attack
oprevious asthma history, atopy, fam hx, use of bronchodilators
PE- acute dyspnoea

IMMEDIATE TX
O2
Chest tube insertion: refer BTS criteria (<2cm air at hilum) etc
CLASSIFICATION
Open VS Close
Traumatic VS spontaneous
Secondary spontaneous VS tension

5. A 29 year old male is brought in by ambulance after being stabbed in the epigastric region. He is
unable to give you more history.
Physical Exam
Vitals: BP - 90/60 mmHg, pulse 135/min, RR - 25/min, Afebrile
General Pale, diaphoretic male writhing on the trolley in pain - 10/10
Neck JVP elevated at 10 cm
Chest Clear to auscultation bilaterally
Cardiovascular muffled heart sounds
Abdo stab wound epigastric but soft to palpation
Extremities weak peripheral pulses throughout, but equal

DDX
Cardiac tamponade
oBecks triad: muffled heart sound, elevated JVP, hypotension

CLASSIFICATION
Traumatic
Non-traumatic: pericardial aetiology (viral, post-MI, uraemia, radiation, CTD)

Haemothorax
Pneumohaemothorax

MX
ECHO-guided pericardiocentesis

Monday, 12/10/2015
15&16.CHEST PAIN [Dr. Spooners re-secheduled version]

APPROACH
CVS ACS
CLINICAL FEATURES ECG TROPONIN
NSTEMI Pressure-like substernal ST depression in 2 or > Raised
chest pain radiating to contiguous leads
the jaw, precipitated by (represent same areas of
stress or exertion, heart)
relieved by GTN or rest T wave inversion
N&V Pathological Q waves
Diaphoresis Axis deviation
STEMI ST elevation of 1mm (chest Raised but doesnt
leads), 2mm (limb leads) matter, ECG decided tx
already
UA Normal (no CHANGES) Normal or raised
from previous ECG records

Aortic dissection
Similar presentation as in ACS
Always consider in all pts presented with cardiac chest pain
Distiguishing features:
Pain: tearing, radiating to the back
Persistent hypotension VS Inferior MI (II, III, aVF) & posterior MI can present with
hypotension too
CXR: widening of mediastinum

Pericarditis
Distinguishing features:
Pain: not commonly described as pressure, > to soreness, pleuritic, < central relieved by sitting
forward
Viral prodorme: myalgia, arthralgia, fever, malaise, sweats
Systemic illnesses: CTDs (RA, SLE)
Younger with no cardiac RFs
Pericardial rub
ECG: Diffused ST elevations (very marked tomb stones appearances)
FBC: neutrophilic lymphocytic (> likely) leucocytosis

Aortitis
Usually present with aortic dissection unlikely to have inflammation of aorta without insult
to media of aorta in first place

Pericardial tamponade
Distinguishing features:
In patients post-trauma
Presented with collapse, hypotension, dyspnoea
Becks triad: muffled heart sounds, hypotension, distended neck veins
Cause
oTrauma
oAortic dissection
oPericarditis
oPost-MI
oMalignancy malignant effusion
oInflammatory: RA, SLE
oDresslers
oFluid overload (CCF, chronic renal disease, chronic liver disease)
oMyocarditis usually with atypical pneumonia
***Look for signs of 1y disease
RESP PE
Triad: Pleuritic chest pain, sudden onset dyspnoea, haemoptysis
Atypical presentation: Post-op multiple small PEs pulmonary HTN HF (symptoms of HF)
RF
ACQUIRED INHERITED
Imobility Thrombophilia
Obesity oFactor V Leiden, antithrombin
Pregnancy mutation, prothrombin, Protein C & S
COCP deficiency
Long haul flights (>4hours)
Illnesses: malignancy, vasculitis, inflammatory
disease (RA, UC, Croohns), chronic renal
failure, underlying cardiac disease
PE: Normal chest expansion, decreased breath sounds, bibasal crepitations, loud P2

Pneumothorax
Sudden onset unilateral dyspnea & chest pain
Hx of previous trauma
RF
oTall thin male, pulmonary disease especially obstructive airway disease (asthma,
empthysema), CTDs, collagen vascular disease, ventilated, cavitating lesions (SA, Klebsiella,
TB), central lines insertion
PE: Tracheal deviation, decreased chest expansion, hyperresonance, small decreased breath
sounds, subcutaneous emphysema

Pneumonia
CAP, HAP
Fever, cough, sputum, SOB, myalgia, arthralgia, fatigue, characteristics features (rusty sputum in
strep penumonia, aplastic anaemia in mycoplasma, headaches, N&V, diarrhea in Legionella)
Chest pain due to pleural inflammation

Status asthmaticus
Dyspnoea, wheezy, inability to talk, light-headed from hypoxaemia, tachycardia, hyper/
hypotensive, cyanotic, pulsus paradoxus (also in tamponade, tension penumothorax,
pericarditis)
Increased epidiameter, decreased chest expansion, hyperresonance, use of accessory muscles,
silent chest, wheeze
Hx of provoking agents
oExercises

Pleural effusion
Unilateral chest pain worst with inspiration & lying on sides of effusion
Symptoms of underlying cause
EXUDATES TRANSUDATES
PE CCF
Pulmonary infection liver disease
RA renal failure
Malignancy thyroid

Malignancy
Hx of COPD, smoking, haemoptysis,SOB, weight loss, cachexia, wasting of small muscles of
hands, Horners syndrome, hoarseness (RLN irritation)
Decreased chest expansion unilaterally, decreased breath sounds, signs of effusion,
lymphadenopathy, Pembertons signs
MSK Costochondritis
Clinical dx
Young, well, no cardiac RF, hx of exertion, exercises
CTDs, autoimmune disorders
GI Upper GI Lower GI
Oesophageal-duodenal Biliary
Oesophagitis: dyspepsia, water brash, Cholangitis
potentially dysphagia Cholecystitis
Achalasia: odynophagia, weight loss Liver
Oesophageal ca: same as above Hep: viral or drug-induced
Bourhaeve: Hx of N&V & haemoptysis
GORD: peptic or duodenal ulcerations Pancreatitis

Splenic
Rupture, infarct

INVESTIGATIONS
Hx & PE
Bloods
oFBC: pL, (thrombolysis, DIC in pneumonia, pleural effusion)
oBlood glucose
U&E: ACS (potassium: predispose to Afib, baseline for angiogram mx)
+GFR
AutoAb screen: pericarditis/ myocarditis, pleural effusion
TFTs: pleural effusion (hypothyroidism), AFib (thyrotoxicosis)
INFECTIO Septic screen: FBC, cultures, CXR
N Urinalysis: Legionella & strep pneumonia Ag
Viral serology; viral pericarditis, pneumonia
Pericardial/ pleural tap
oC&S: microorg presence, pH: low in exudative causes, glucose, LDH: malignancy, cytology:
malignant cells
CVS ECG
CXR

Cardiac biomarkers: troponin, BNP (lots of pathology that can cause HF)
ECHO
oACS: hypokinetic wall motion (RMWA)
oaortic dissection: dilated aortic root
otamponade: fluid
oPE: R heart strain
omyo/pericarditis: fluid around heart muscles, indiscrete borders
Angiogram
oSTEMI (primary angiogram: up to 6hours)
oNSTEMI, aortitis & aortic dissection associated with coronary artery disease
RESP CXR
CT thorax: effusions

D-dimer: PE (useful negative predictive value)


CTPA

ABG: if dyspnoeic and desaturating, T1RF with respiratory alkalosis


PEF: asthma
Spirometry: not in acutely unwell pts, usually in CF pts (chronically decreased lung fx, changes
of symptoms with exacerbations)
GI OGD
Barium swallow
Gastrograffin enemas
CTAbdo

Monday, 28/9/2015
17.FEVER IN THE NEUTROPAENIC PATIENT
CASES
1.A 48 year old man presents to the ED with fever (>38C), weakness, and cough. He has been suffering from
shortness of breath and night sweats in the past 10 days. He has had diarrhoea for the past 5 days and lack of
appetite, but he denies any possible food poisoning. He has scratch marks on his arms and legs. He has a history of
alcohol abuse, and has been smoking 20 cigarettes a day for the past 25 years. His blood count reveals an ANC
(Absolute Neutrophil Count) of 1000/L [low]
Symptoms Aetiology
Fever (>38oc), URTI
weakness, and cough TB: if +haemoptysis, +wt loss, +exposures, +foreign travels
Diarrhoea for the past viral GE infection
5 days and lack of bacterial GE: salmonella, shigella, campylobacter, E.coli. Ask if on Ab, any recent hosp
appetite admission
Scratch marks on his itchy (pruritus): cholestasis (Gilbert syndrome: +jaundice, +infection)
arms and legs uraemia
scabies (rare, in between web soaces of fingers & toes)
Alcohol abuse +duration, +relationship to itching +CAGE? (not appropriate in acute settings!)
liver chirrosis
Smoking 20 cigarettes direct effects: explaining cough & SOB
a day for the past 25 exacerbation of COPD
years bronchial malignancy
pneumonia
bronchiectasis
WORK UP
FBC
o WCC (leukopaenia): HIV (check for CD4 count), bone marrow dysplasia
MCV, MCH:
o Macrocytosis: anaemia or Folate/ B12 deficiency (common in alcohol abuse)
o anaemia of chronic dzs
o microcytic: anaemia from bleeding chirrhosis of liver (cause varices, coagulopathy: low pL due to portal HTN
exerting back pressure on spleen leading to thrombocytopaenia)
Cell count differential
RFT: hepatorenal syndrome
LFT: AST, ALT, ALP, bilirubin +albumin +GGT
U&E
o dehydration, hypokalaemia (low amplitude T waves), Cl, Mg, Na, Ca (low electrolytes precipitates arrhythmias!)

INFECTION BONE MARROW (BM) DISEASE


CRP, ESR BM aspirates
Blood cultures BM trephine biopsy: very sore!
o from peripheral veins or central lines (CVC, PICC lines) Blood fllm: blasts (immature)
o fever (>38oC )
o sample: sputum, bronchoscopy CXR
sepsis CT TAP
Urine culture: MSU Bronchoscopy
Stool culture Viral screen: hep, HIV
o regular: E.Coli, salmonella, shigella, C.bacter
o ask specifically if for C.Diff
line/ drain swab
ascitic tap if has ascites (+fluid thrill, +shifting dullness)
o subacute bacterial peritonitis (SBP)
+/- CSF
o sepsis
o LP only if neurological symptoms present

DDX
CLD
TB: pulmonary or disseminated
Lymphoma; +fever, +night sweats, +splenomegaly & lymphadenopathy on exam
HIV
Multiple myeloma: +high ERS, +renal failure
Carcinoid tumour: +SOB, -diarrrhea, +facial flushing (common with R-sided heart murmur
2.A 34 year old woman presents to the ED as a referral from her Oncologist with fever (38.5 C), fatigue, anorexia,
nausea. She was diagnosed with Acute Myeloid Leukaemia a month prior. She had her first cycle of chemotherapy
ending 10days ago. She has no previous medical history, she doesnt drink and she quit smoking 5yrs previously.

Fever (38.5 C), fatigue, anorexia, nausea


infection post-chemo
post 10days BM suppression sudden WCC, pL & Hg drop

+ask about headcache, earache, vision ,rashes, neck pain


+chest pain, dyspnoea, new murmurs (IE)

DDX
AML (M = neutrophils)
o neutropaenia
o symptoms of anaemia (chest pain), recurrent infections, palpable LNs, bleeding
o blood smear: auer rods
o immature blasts neutrophils although of high neutrophils number (useless!)

ALL (L = lymphocytes)
o lymphocytopaenia
o blood smear
WORK UP
FBC- monitor changes in counts
U&E- elevated urea (dehydration), electrolytes down :also detect renal impairment
LFTs- cholecystitis, ascending cholangitis
Amylase- pancreatitis
Blood cultures

TX
Empiric Ab for febrile neutropaenia
o tazocin 4.5g TDS IV (reduce to twice a day regimen in pt with renal dysfx GFR<15)
o + gentamicin (based on weight: 3-5mg/kg OD IV)
IV fluids
Paracetamol
o giving antipyretics in febrile neutropaenic pts: can mask the temperature!
o codeine or morphine-based [non-antipyretics analgesia] not to mask the temp
Electroytes replacement
Dextrose + saline / Hartmanns
Anti-emetics
o Stemetil (Chlorpromazine) PO (N&V!) IV
o Ondansetron (usually reserved for chemoT pts- work centrally on brain not on gut)
o CI to IM Stemetil if at risk of coagulopathy (can drop Hg! Use IV anti-emetics)

18.APPROACH TO SKIN LESIONS

APPROACH

Qxs to ask
HPC OTHERS
Ask pt him/herself what might be causing the lesion PMH: chronic dzs
Painful: Herpes zoster (+dermatomal distribution) oHIV Kaposi: lesion on forehead does not
VS non-painful blanch with pressure
Itchy: psoriasis, contact dermatitis (+border well oScleroderma: tight skin on back of neck)
demarcated outlining the contact area, +allergy, o***HIV: ask when was diagnosed, sex life for
+hairdressers), or not STDs, drug abuse
Insect bites Meds: had same lesion before? What meds taken?
Location Worked or not?
Duration Allergy: to pets, etc
Evolution: to rule out malignancy (bleeding,ulcer, Change in job
pigmentation or depigmentation, speed of growth) Surrounding people with similar symptoms
Exacerbations Change of shower gel
Sensation
Systemic symptoms
oneurological: neurocutaneous dzs

Exam
INSPECTION PALPATION
general Tenderness
ocolour: jaundiced, bluish Fixed or mobile: fixed (non-inflammatory,
head malignant)
ohair loss areas, lesion on scalp Polarised light: used to see pigmentation of skin (in
face dark environment!). Ash leaf pattern in tuberous
ospots or lesions in mucous membrane in eyes sclerosis
okopliks spots in measles (usually in upper Dermatoscope: visualize subsurface lesion
palate) in mouth
oulcers or hair loss in nose
gluteal cleft & anal region
lymphadenopathy: neck, axilla & trunk

Lesion description
Number: single or multiple
Distribution: dermatomal (Herpes Zoster), generalised (drug reaction)
Arrangement/ configuration: patterned (Herpes Zoster)
Size: x cm
Colour: skin-coloured, hypo/hyperpigmented, reddish/ erythematous
Shape: symmetric/ assymmetric, raised or non-raised
Border: well demarcated (contact dermatitis)
***Melanoma: ABCDE

Definition
Macule non-palpabel, a flat circumscribed area of skin Vesicle fluid filled lesion within or below the
color change epidermis, less than 0.5cm in size
Papule a raised circumscribed lesion less than 0.5cm in Pustule blister with pus filled lesion of any
diameter, isolated size
Plaque a raised lesion on the skin that is more than 2cm Bulla a fluid filled lesion that are larger than
in diameter that can be formed from the confluence of 0.5cm in size (pl. Bullae)
papules and nodules. For lesions between 0.5 and 2cm in Telangiectasia small dilated blood vessels on
diameter these can be called small plaques. In groups skin or mucosal surface
Patches: palpable plaques & >2cm? Purpura haemorrhagic spots >2mm
Nodule a visible and palpable raised solid mass in the diameter, does not blanch on pressure
skin, more than 0.5cm in diameter. It can be epidermal and Palpable purpura palpable petechiae that
dermal, dermal and subcutis, or just subcutis. Can be made are red well circumscibed maculopapular
of fluid, inflammatory, neoplastic or extracellular material lesions with an area of central haemorrhage

Systemic Disease With Cutaneous Findings


DM: scleroderma
Thyroid: hyperpigmentation, dry skin (hyperthyroidism)
Addison: Bronzes skin
SLE: malar rash
RA: pyoderm agangrenosum
Dermatomyositis: Gottrons nodules (on extensor surfaces, blue-violet)
Infections: Herpes Zoster, endocarditis (Janeway lesion, Osler nodes, Roth spots)

CASES
1. A 47 year old man presents to the Emergency Department as a referral from his General Practitioner.
He has several skin lesions on both his legs and one on his nose. The lesions are purplish maculae
and papulae, and some look more like small ulcers. His left leg looks swollen also. All his symptoms
started 4 weeks before. He has recently been abroad (Argentina). He is HIV positive and is also a
diabetic.
DDX
Kaposi sarcoma

2. A 26 year old woman presents to her General Practitioner complaining of a burning pain in the left
side of her back. At the examination she presents a rash characterised by erythematous papulae and
vesicles. She also has some vesicles on her left hip. She hasnt been abroad recently and she is on
antibiotics for a recent otitis.
DDX
Shingles

3. A 76 year old man presents to his dermatologist for his annual skin mapping. While examining his
back the dermatologist notices a dark brown asymmetric lesion of 8mm of diameter. He has a
history of non-small cell lung cancer and he had a lobectomy 5 years prior.
DDX
Malignancy: malignant melanoma (<1.5mm = 90% 5yr survival. >3.5mm = 35% 5yr survival)
Metastasis
Non-malignant: melanocytic naevi

Friday, 2/10/2015
19.BACK PAIN

APPROACH
Classifications
Acute VS chronic

MSK
MECHANICAL NON-MECHANICAL
oDisk herniation o Haematoma
oOA, OP
oSeronegative arthropathy (ankspond, psoriatic, reiter syndrome, enteropathic)
oAtypical RA
Visceral
GI RENAL GYNAECOLOGICAL
oAAA, pancreatitis, perforated orenal pathology oprostate, PID, endometriosis,
PUD, liver dzs, GB abscess, calculi, AVM, malignancy ovarian ca, large ovarian
cysts
Neurological
oUMN signs: spinal cord involvement only
oUMN + LMN signs: spinal cord + peripheral nerves

Pathothysiology and aetiology


CNS (Damage to AHC AND above: UMN) Peripheral (Damage to below AHC: LMN)
Spinal stenosis narrowing of spinal canal Spinal stenosis
Disk herniation Trauma
SOL: haematoma, malignancy EX: hemitransection at L1
Trauma oLMN of sensory level of L1 (bilateral paraplegia,
Infections reflexes N)
Encephalitis/ meninigitis (transmitted via dura in oSpinal cord below L1 (UMN)
conus medullaris manifested as back pain)
Cauda equina syndrome: usually caused by causes
above & inflammatory disorders

Malignancy
Infections
oTB, spinal abscess, bone abscess, osteomyelitis, transverse myelitis, HIV, brucellosis, Hepatitis
Psychological: somatisation, depression, anxiety
***one cause can be secondary to other causes!

Red Flags
Patient Age: <20, >50
factors Metastatic malignancy
Known spinal cord lesion
Known congenital abN of spinal cord (spina bifida, severe congenital kyphoscoliosis)
Know MSK abN of spinal cord
Symptoms Weakness (rapidly progressive)
Pain: intractable, thoracic (unusual!- transverse myelitis, spinal abscess)
Saddle anaesthesia
Faecal/ urinary incontinence (saddle abN)
Autonomic dysfunction
Neurogenic claudication: back pain worse with ambulation / walking (mass lesion, abscess
compressing lesion aggravates pain with flexion), cannot stand up straight
Bilateral
Signs Weight loss
UMN lesion findings (LMN lesion not as worrying involve only single nerve)

CASES
1. A 53 year old man presents to his GP complaining about acute back pain after lifting a dishwasher
from the back of a van. He has never had similar episodes previous to this one. He has a productive
cough and he reports having had a temperature a couple of days before. He has a history of Type 1
diabetes and he had his gallbladder removed 4 years ago.
DDX
Infection miliary TB
Mechanical hx of recent trauma
Malignancy mets, lymphoma, leukaemia, MM, 1y bone malignancy, spinal carcinomatosis (mets deposit on spinal cord
fr a 1y somewhere else)
**Diabetic myotrophy in diabetics, but not central usually peripheral
osteoporosis- vertical collapse
myeloma

Most probable
INFECTION MALIGNANCY
TB: night sweats, weight loss, RF of exposure (crowded area, Lymphoma: bimodal age distribution (not in
poor vaccination, endemic region, etc) favour of the dx), normal WCC, pL, RCC but low-
Osteomyelitis: hard to dx, previous orthopaedic surgeries, functioning (get infections, bleeding, etc)
trauma to the bone, previous cellulitis, underlying diabetes haematological malignancies
HIV: > symptoms: loss of weight, systemic abN, RF, longevity of o look for symptoms of anaemia: SOB, syncope/
illness (seroconversion of illness), recurrent infections (fungal pre-syncope, palpitations, mucosal bleeding
nails, URTI) due to low pL (menorrhagia, gingival, bruising,
Transverse myelitis: not just pain, + focal neurological deficit of petechia, haematuria)
spinal cord (UMN signs), prodormal viral illess (arthralgia, Prostate: usualy would have presented with
myalgia, low grade fever) symptoms of BPH + weight loss
Spinal abscess: red swollen area on back
Brucellosis: farmer, drink pasteurized milk
Hepatitis: preceeding illness (jaundice), exposure

INVESTIGATION
Xray: lumbosacral spine abN ****If infectious aetiology is most likely:
FBC: pancytopaenia Extended septic screen: high vaginal & penile swab, viral serology, CSF analysis,
(haematological malignancy) ECHO, bone scan
Blood film: always follow FBC ****If other causes > likely:
ESR & CRP: raised in all causes! URGENT MRI
Septic screen: Cultures (blood,
sputums, wound, urine, faeces, Immunomarkers:
nasal swab), CXR, FBC oANA, RF: non-specific
oHLA-B27: Ank spons
oPsoriatic: biopsy hyperkeratinisation
oReiters dzs: urethritis, arthritis, uveitis due to STIs (Chlamydia),
Campylobacter pylori preceeding GU illness

2. A 24 year old man presents to the Emergency department with a dull pain in the lumbar region,
stiffness and pain in his knees. He has had these symptoms for 2 weeks and has never suffered from
back pain before. He also has a red eye and complains of blurred vision for the last 2 days.
DDX INVESTIGATION
MSK: ank spond Serology
Infection < likely Spinal Xray: lumbar region
IBD

3. A 63 year old woman presents to her General Practitioner complaining of acute back pain after
working in her garden. She had few episodes a couple of years ago, when she was still working as a
waitress in a restaurant. She is a current smoker and she has COPD
DDX INVESTIGATION
MSK Chest Xray
o Vertebral fractures secondary to OA [RF: smoking Mammogram
post-menop, on steroids long term for COPD] Back pain + anaemia= myeloma
o Trauma
o Muscle spasms
Malignancy
OA
Cauda equina
Somatic

Red flag:
WL
TEMPT
CANCER
AGE; YOUNG/OLD
20.The Acute Swollen Joint
APPROACH

Aetiology
INFECTIOUS Septic arthritis
oSTI: Gonorrhea, Chlamydia
oCampylobacter
oReiters: STIs, Campylobacter, Salmonella, Shigella, E.Coli
oLyme dzs
oTB
oOrthopaedic procedures, IVDU
INFLAMMATORY Seronegative arthropathy
oSacroilial, large joint arthritis
RA
oBilateral symmetrical small joints
Mixed connective tissue dzs
oLupus, vasculitis, sarcoidosis
METABOLIC Gout
oHyperuricaemia
oChronic renal failure
oDrugs: cyclosporin
Haemochromatosis
oGenetic dzs of high Fe levels
Pagets dzs
oAbN destruction of bone & abN reconstruction of bone deformed
TRAUMA Haemoarthrosis: bleeding diathesis, iatrogenic (anti-pL, anti-coag)
FUNCTIONAL Fibromylagia

Clinical Features
PATIENT FACTORS SYMPTOMS SIGNS
Hx of trauma Fever Hyperuricaemia (gout, tumour lysis
Hx of exposure to infection Loss of function syndrome)
(Lyme) Weight loss (infectious, Extra-articular manifestations
Travel hx- Salmonella, Lyme enteropathic. Not so much in Rash (bulls eye, erythrema
Sexual activity STI Pagets, vasculitis, seronegative) migrans), migrating arthritis
Alcohol gout Haematuria Stoma bag: enteropathic
Diet: cheese, red meat gout Seizures, strokes Livido reticulosis: SLE
Meds: anti-coag, anti-pL

INVESTIGATIONS
Synovial fluid aspirate: beneficial in infectious, gout
o Analysis:
gross appearance (blood, cloudy)
microscopy, C&S
viral serology
anti-leptospirosis Ab (Lyme)
WCC
Glucose (also do serum glucose compare: expected to be lower than serum utilized by bacs)
Crystals: urate or calcium pyrophosphate (pseudogout) [gout negatively birefringens, pseudogout-
positve]
Coag profile: inherited coagulopathy
WCC, pL - infection, Hg haemoarthrosis
AutoAntibodies
o1 in 200: need to dilute the solution 200x to detect Ab pt symptomatic or x, if asymp no further testing
o1 in 2000 or >: dzs > likely, +ve
oANA: if ve no point ordering for > specific autoantibodies
oIf ANA +ve:
SLE: anti ds DNA, anti histone
Joint Xray
OA -LOSS SLE HAEMOCHROMATOSIS PAGETS
loss of joint space Synovial Similar to OA: AbN architecture of
subchondral sclerosis swellings degenerative bones
subchondral cysts
osteophytes
MRI & bone scan
oIn septic arthritis look for ascending osteomyelitis
osurgical intervention (joint replacement if found)

TX
INFLAMMATORY/ PSORIASIS OR
HAEMOCHROMATOSIS RA
INFECTION ENTEROPTHIC
Antibiotics Fe levels, transferrin level saturations Pain relief Tx underlying cause
Analgesia Genetic test DMARDs: steroids, Pain relief
+/-PhysioT Venesection on regular basis methotrexate PhysioT
+/-Splinting Fe-chelating agents Failure of one or >
Pain relief DMARDS
PhysioT biological agents

CASES
1. A 70 year old man presents to the Emergency department with a severe pain in his left knee. The knee is swollen
and red. He has a history of Type 2 Diabetes which is well controlled with metformin. He reports he was on a hike
one week before and he fell on a slippery trail.
DDX
Septic arthitis
Haemochromatosis: Fe deposition in pancreas may cause secondary DM [Tx depends on aetiology (ex: CF- insulin
deficient)]

2. A 34 year old woman presents to the Emergency Department with a 1 day history of a swollen, tender,
erythematous left wrist. She also reports fever and cough; she came back from a camping holiday in Croatia one
week ago.
DDX
Lyme disease
o Itchy painful rash: eryhthema migrans (bull-eye lesion), salmon red colour, armpit, groin, back of knee, belts line
o Viral-like
o Facet joint points
o Myocarditis, heart block
o Neurologic: meningitis
o Borelli Burgdoferri, tick vector
o ELISA
o Tx: doxycyline

3. A 29 years old woman presents to her GP complaining about palpitations and swollen, sore ankles since the
week before. She has a history of recurrent UTI and a mitral valve prolapse. She also reports that she has attended
the ED 3 days before for possible food poisoning
DDX
Reiters dzs
o preceeded by gu/gi symptoms
o uveitis, iritis, scleritis
o UTI
o arthritis
o Salmonella, Shigella, C.bacter
INVESTIGATION
Stool culture
Serology: RF r/o RA
ESR/ CRP
Genetic studies: HLA-B27
Synovial aspirate (arthrocentesis): increased neutrophils + PCR
Imaging: CXR (exclude other causes of arthritis) ,US joint, synctigraphy (radionuclide bone scan)
Monday, 5/10/2015
21.PYREXIA OF UNKNOWN ORIGIN

APPROACH
Criteria
o Fever >38.3
o Lasted >3weeks
o 2weeks of basic levels of investigation done (septic screen)

Aetiology
INFECTION Bacterial Viral Fungal
TB EBV Candida
RF: Benign in most ppl Oral thrush
oTravel hx to endemic Can be life-threatening in oAsthmatics inhalers
countries (no BCG others containg corticosteroids
vaccination) CMV GI candidiasis
oDense area with poor Immunocompromised oIn immunosuppressed pt
sanitation (India) (HIV, chemoT) oPUO
oWeight loss, HIV Candida septicaemia
haemoptysis, cough, Prodrome seroconversion oIn patient with lines
night sweats, abd pain illness infectious phase (renal, chemoT)
(appendix- cause (resp, nail, etc) Aspergillus
appendiceal mass RFs in hx: ABPA
causing obstruction, orisky sexual activity Chronic tiredness
palpable mass seen in (MSM, multipartner, Dry cough & wheeze
CT abd), jaundice (if unprotected sex, sex worsening of cough in
affect liver) workers, sex with early morning
Borrelia (Lymes dzs) carrier) Investigation: IgE levels
Rash associated with oblood transfusion
fleeting arthralgia oIVDU
(monoarthritis) oVertical transmission
Flu-like symptoms Hepatitis
AV conduction block B&C
Severe headache Herpes
secondary to encephalitis Sympt: vesicles
Salmonella/ C.Bac/ Shigella Can be asymp: herpes
Severe diarrhea, weight loss encephalitis, spinal herpes
C.Diff
Usually lower abd pain
Prev exposure to Ab
(macrolides &
fluoroquinolones
particularly)
Do toxin assay
Legionella
Water contamination
(shower)
MALIGNANCY Lymphoma
Fever, wt loss, sweats, anorexia, lumps (cervical or mediastinal LNpathy)

INFLAMMATORY Crohns
Triad: crampy abd pain, anorexia ,wt loss
UC
Crampy diffused abd pain & diarrhea
Sarcoidosis
Elderly, white & black
Resp symptoms predominantly, fever, chronic fatigue, dyspnoea on exertion, dry cough
Bihilar LNpathy
Hypercalcaemia: abd pain from pancreatitis, constipation, peptic ulceration
Neurosarcoid: seizures
Indiscrete maculopapular heals with bruising rash: eryhthema nodosum
Amyloidosis
Vasculitis
Giant cell arteritis
o>60, female propendorence
oHeadache, scalp tenderness, jaw claudication, visual acuity impairment blindness
oPreceeding polymylagia rheumatica: pain/ weakness on shoulder or pelvic girdles
years before gets GCA
oCan have Gower sign +ve
SLE
oMalar rash (erythematous flat rash in sun-exposed area)
oArthralgia, myalgia, fever
oRenal dzs: haematuria, oliguria, acute renal failure
oMobile arthropathy (progression of arthralgia)
oAbd pain: from peptic ulceration
Mixed CTD
oDysphagia + epigastric pain
Systemic sclerosis
oCREST
Polyarthritis nodosa (PAN)
oHepatitis c infection
oRash livido reticularis (purple)
oRenal features

CASES
1.A 54 year old man presents to the emergency department as a referral from his general practitioner
with a 6 week history of fevers. He complains of malaise, anorexia and epigastric pain. He reports loose
stools, gas and is passing mucus per rectum. He suffers from hypertension for which he takes an ACE-
inhibitor, and he denies having taken any recreational drugs. His only recent travel was 8 weeks
previously when he went to visit his family in Ecuador.

DDX
INFECTION MALIGNANCY INFLAMMATORY
Amoebeasis Lymphoma: if in Sarcoidosis
Aspergillosis older pt oage < likely, epigastric pain from pancreatitis (hypercalcaemia)
Vasculitis:
oGCA < likely

2.A 23 year old university student presents to the emergency department as a referral from his general
practitioner. He complains of fevers over the past 8 weeks, recurring on a daily basis, often with two
spikes of temperature over the course of the same day. He also complains of fatigue and arthralgia in
both of his elbows. On examination a salmon coloured rash is noted on his back. He has not travelled in
the last 6 months and he doesnt recall being unwell recently aside from a sore throat 4 weeks
previously.
DDX
INFLAMMATORY INFECTION MALIGNANCY
AOSD: Adult Onset Stills disease Streptococcal infection Lymphoma
fever, joint pain, salmon- Pharyngitis Lumps & bumps (cervical
coloured raised rash rheumatic fever (JONES: myalgia, LNpathy)
undulating fever arthralgia, chorea, fever, Leukaemia
Vasculitis myocarditis causing chest pain Preceeding URTI, tiredness,
SLE and palpitation, emboli) arthralgia, myalgia
PAN post-strep glomerulonephritis Bruising, gingival bleeding,
IgA nephropathy haematuria
oliguria, haematuria, Recurrent infection
hypotension, oedema if nephrotic Anaemia (if Hg <5) angina

3.A 68 year old man presents to the emergency department as a referral from his general
practitioner. He complains of a 3 week history of intermittent fever (37.5-38C) which did not respond to
antibiotics given by his GP. He complains of hoarseness and reports that he has lost weight in the last
couple of months, although he feels his abdomen is slightly swollen. He has also had some episodes of
headaches and back pain which were not well controlled with NSAIDs. On clinical examination he is
noted to have scrotal varicoceles and lower extremity oedema. He has a history of hypertension and has
recently been to Mexico to visit his daughter

DDX
INFECTION MALIGNANCY
Subacute meningitis or encephalitis METS
Subacute osteomyelitis oRenal causing back pain and testicular swelling
Endocarditis : was on Ab before oOropharyngeal/ lung causing hoarseness
oMets to liver causing ascites
Non-Hodgins dzs
oAdenopathy causing the symptoms

INVESTIGATION
Bloods Viral serology
oESR, CRP [to monitor therapy , do every 2-3days] oMonospot for TB
Septic screen oHIV, Hep
oculture everything, CXR, FBC PCR
Imaging Anti-legionalle Ab
oCXR CSF analysis
oMRI brain
oCT scan
oTOE
MALIGNANCY INFLAMMATORY
Blood smear, bone scan, BM aspirate (for AutoAb: ANA, if +ve do the specifics autoAb
haematological malignancies), LN biopsy (FNA or Sarcoidosis: cytology from LN biopsy, ACE levels,
core for lymphoma) CXR

TX
Antimicrobial
IFN
Sarcoid: symptomatic with/ without steroids
Lymphoma: chemoT based on genetic profiling

22.COMPLICATIONS OF CHEMOTHERAPY
1. A 29 year old man, newly diagnosed with a diffuse large B-Cell Lymphoma is undergoing his first
cycle of chemotherapy. On day 3 of his admission he develops nausea, vomitting and muscle
cramps. His blood results show uric acid 672umol/L (normal= 110-420umol/L), serum creatinine
205umol/L (normal = 50-120 umol/L), calcium 1.85mmol/L (normal= 2.2-2.63mmol/L) and
phosphate 2.03mmol/L (normal = 0.65-1.62 mmol/L).

chemo: drug that target dividing cell


B symptoms of lymphoma: fever, WL, NS
Pathologist: mitotic count, sensitivity to chemo: grade
DNA; purine broken down to uric acid which transfer to kidney, electrolytes (inside cell- K, phosphate, purine
and pyrimidines, outside- Na, Cl)
Phosphate crystallate with calcium=ca phosphate, build up.

Potassium: 7.5 (high)- can cause arrythmia


-insulin hide the postassium for a while.
-diuretics (loop)- take time to work
-do ecg, give calcium gluconate (do nothing at all at the K but it is cardio protective)
-long twave, qrs-wide,
-salbutamol cause hypokalaemia- nebulised
-put on telemetry

high creatinine;
-monitor urine output, 30ml/hour

uric acid- cause gout


more worry about the kidney, tx: allopurinol, RPSK
causes high uric acid: gout, chronic kidney dz, tumour, diabetes (renal damage), dehydration

before start pt on chemo: hydrate using fluid, allopurinol

CASES

2. A 52 year old woman with a diagnosis of ovary cancer presents the oncology department for her second cycle of
chemotherapy. On day 2 of of the chemotherpay cycle she develops profound diarrhoea, she did not experience this
symptom during her previous cycle. When asked about her recent medical history she says that she developed a UTI last
week and was commenced on ciprofloxacin.

Chemo affect alimentary cells as well, bone marrow, causing malabsorption


c.diff
neutropenia

DDX
Mucocitis (mucous membrane inflammation)
ogastritis, oesphagitis, colitis, etc
oIn this pt: pancolitis (diarrhea)
oRadioT agents radiation enteritis or colitis in several years later [strictures or adhesions in imaging or
symptoms of bowel obstruction]
C. Diff
Faeco oral transmitted infection
Penicillin allergy
oImmediately: rash, anaphylaxis, angioedema, bronchospasm
o2-3days after exposure: abd pain, diarrhea, N&V

MX
IV fluids
NG tube rest bowel
PPI
Steroids? Controversial >prone to infection, works for nausea (dexamethasone)

C. DIFF (specifically) INFECTIOUS


Stool toxin assay IV fluids
Fluid rehydration: oral metronidazole or IV Rest bowel
gentamicin Antimicrobial agents
Do not give chemoT while on ACTIVE infection oE.Coli: HUS risk (spikes temp, not stabilised)
Anti-diarrheal agents
oCan prolong infection, in C. Diff pt give massive risk
of toxic megacolon and perforation

2.A 78 year old man is brought into the emergency department with nausea, vomiting and fever. His
GCS is 12 (E3, V4, M5). He is day 4 post chemotherapy for his lung cancer; his wife reports he
experienced a progressive weakness in the past few days and lack of appetite. He has been recently
diagnosed with Type 2 diabetes and is taking metformin. Bloods are taken show neutrophils of 0.8 x
109/L (2.5-7.5 x 109/L)

DDX of febrile neutropaenia


CNS toxicity
Brain mets
Lactic acidosis secondary to metformin overdose
SIADH
PTH-releasing small cell lung ca
oCause hypercalcaemia

MX
FIRST: Broad spectrum penicillin or beta-lactam Ab tazocin + aminoglycosides

Investigation
Septic screen
CT brain BEFORE LP (exclude SOL)

MX
INITIAL
Specific Ab: vancomycin- cover MRSA
Steroids?
ocerebral oedema from mets
oin pt given vancomycin, do not give steroids reduce absorption in BBB
Aggressive fluid mx
Catheter urine outputs
***If not improving add anti-virals still not improving anti-fungal

ChemoT beginning with Cs or Ds for high risk


oSE: alopecia

TX TO COMBAT CHEMOTHERAPIES SEs


Anti-emetics
oS5-HT3 antags: Ondansetron a day before start chemoT
oPro-kinetics: metoclopramide - malignancy or mets that can compress on bowel wall
Anorexia
oSteroids: dexamethasone
Immunosuppression effects
oProphylaxis Ab
oMouth washes: anti-fungal (nystatin prevent candidiasis)
oLeukopaenia
Granulocytes colony stimulating factor (GCSF): Increase WCC counts once a week or every 3-4days
oAnaemia
CMV -ve irradiated blood transfusion (high risk of CMV infection, transplant candidate- hypersensitivity
rxn, rejection, infection reactivation)
Eryhtropoeitin
opL replacement: transfuse if <50

Friday, 9/10/2015
BLEEDING/CLOTTING DISORDERS

Bleeding
PLASMA PLATELET
Haemophilia A, B, C ITP: Idiopathic thrombocytopaenic Purpura
VwB HIT: Heparin-Induced Thrombocytopaenia
TTP: Thrombotic Thrombocytopaenic Purpura

Clotting
ACQUIRED INHERITED
Pregnancy Factor V Leiden
Immobility Anti thrombin deficiency
OCP Protein S & C deficiency
Prothrombin mutation

CASES
1. A 38 year old lady presents to the Emergency Department complaining of a three day history of band
like severe abdominal pain, which is worse post-prandially. On examination the abdomen is soft with
diffuse tenderness on palpation, worse in the right upper quadrant. Past history includes an upper limb
DVT following dental extraction under general anaesthesia. Medications include a pregnancy
supplement, as she is trying to conceive at present. An abdominal ultrasound reveals a large portal vein
thrombosis.
Further Qxs for:
Predisposing factors
Fam hx
Provoked DVT Unprovoked DVT
Long haul air travel UL DVT following dental extraction
Recent surgery o last 15mins, not immobilized
Meds : COCP, HRT, estrogen does not provoke the
Pregnancy thrombosis
Sub-therapeutic anticoagulation

Mechanisms of portal vein thrombosis causing post-prandial pain


Thrombosed vessels dilatation due to O2 demand for digestion causes pain
Non-occlusive: venous
Occlusive: arterial HTN, critically ill pt (vascoconstrict- relative hypotension mesenteric ischaemia)
Thrombotic events in SMA/V
Venous stasis 2y to PVT mesenteric ischaemia

DDX
INHERITED ACQUIRED AUTO-IMMUNE
Factor V Leiden: wont have APL syndrome: previous miscarriages, SLE
issues with pregnancy unprovoked DVT (underlying clotting probs)
Protein S & F deficiency Malignancy: myeloproliferative disorders
Prothrombin mutation ITP: pL disorder, < likely
Antithrombin deficiency Thrombocytosis

Indications for screening


Recurrent VTE
Thrombosed in unusual sites
Recurrent miscarriages: >3miscarriages <10weeks gestation

Who is > likely to develop VTE


IBD patients: pelvic VTE
Nephrotic syndrome

INVESTIGATIONS MX
FBC: Hg, pL LMWH
Blood film & smear oin pregnancy
LFTs oprophylactic enoxaparin (Clexane) BD subcut
Coags: pT, aPTT, thrombin time dosing based on actual body weight
Factor V literature not clear on dosing in obese (>120kg) or those
Protein C & S with mild renal impairment
Activated Protein C resistance oprophylactic tinzaparin (Innohep) 175IU/kg OD subcut
Anti-cardiolipin Ab & lupus anticoagulants Warfarin: if not conceiving, do INR baseline, INR after 2nd day,
D-dimer monitor daily afterwards
Homocysteine levels: homocystinuria NOAGS [Dabigatran, Apixaban, rivoraxaban]: no monitoring
needed

2. A 21 year old man presents to the ED complaining of a one day history of a painful swollen right knee
with no antecedent trauma. He reports a similar presentation last year where the same knee was
affected. A diagnostic joint aspiration reveals a haemarthrosis (bleeding of synovial vessels)

Symptoms & signs to be screened


Spontaneous bleeding into joints or muscle beds Blood tranfusion: have u had any, how many
(quads-compartment syndrome) can dev antibody, renal transplant candidates,
Dental extraction allergic rxns, HIV, blood-borne dzs
Menorrhagia Meds: aspirin, Ab (b-lactams)
Unexplained epistaxis Haematuria: not indicator of renal fx
Escessive petechiae Malaena: > worrying than haematuria
Bleeding after vaccination or phlebotomy
***if patient presented with haemoarthrosis on established bleeding disorder do not do joint aspirates
can cause infection & bleeding do US instead

DDX
Haemophilia A (8)- most common, B (9), C (11) Myelodysplastic disorder
o Mild: 5-40% normal activity Heparin-induced thrombocytopaenia: common in
o Moderate: 1-5% normal factor activity vascular dzs, cardiac surgery requiring bypass
o Severe: <1% normal factor activity Dietary Vit K deficiency
symptomatic in early years of life Liver dzs
VwB DIC: usually symptomatic, too much bleeding &
Factor V Leiden clotting at the same time
ITP Amyloidosis
TTP Long-term use of B-lactams: work on pL pathway
**PL bleeding: immediate bleeding VS haemophilia: delayed bleeding

Investigation of underlying bleeding disorder


As outlined above

MX
Replace deficient factor
Red cells
FFP
Fibrinogen
Tranexamic acid: dental bleed, epistaxis, pts on warfarin

Complication of bleeding disorder


Blood transfusion Ses
Intracranial bleed intracranial nerve palsy: neurological deficit, haemorrhagic stroke
Increased MR
Menorrhagia
Gi bleed
Exsanguination
Genetic risk and counselling

24.ANAEMIA

DEFINITION
Decreased hemoglobin <13.5g/dL (men) or <12g/dL (women)
OR
Decreased hematocrit <41% (men) or <36% (women)

Anaemia symptoms:
Presyncope
Chest pain
Palpitation
Respi: dsypnoea
Fatigue, malaise
Examination:
Pallor
Postural drop in BP
RR increase, HR increase
Hands: pallor of palmar creases
Eyes; pallor of conjuntiva

Reduce production: iron,

vit b12- pernicious anaemia (intrinsic factor), healthy stomach, reasonable diet, need acid (long term PPI),
coeliac (malabsorption), Crohns, surgery to remove terminal ileum

folate: pregnancy, malabsorption-coeliac crohns surgery, methrotrexate, diet alcohol , AED, cancer

healthy marrow, hormones- epo (CKD), functioning thyroid

Unhealthy marrow: benign- genetic/congenital (aplastic anaemia), acquired


Malignant- primary/ mets

Increase loss:

CASES
1. A 66 year old man is admitted to the Emergency Department following a collapse, on a
background of nausea and vomiting. On review of systems he reports a one month history of
fatigue. Past medical history includes hypertension which is well controlled on medication.

His blood tests reveal


Haemoglobin 8.4 g/dL (11.5 16g/dL),
creatinine 60 mol/L (50 - 120mol/L),
B2 microglobulin 3.1mg/L ( <2.4 mg/L), -positive in multiple myeloma and amyloid
albumin 28g/L (35 53 g/L).

DDX
Liver disease: decreased albumin, +hx of alcohol excess
Nephrotic syndrome: decreased albumin, fatigue, -N Cr, +oedema
Multiple myeloma: +high B2 microglobulin
Lymphoma, leukemia, MGUS (Monoglonal Gammopathy of Unknown Significance), amyloidosis: also cause
high B2 microglobulin
Malignancy: colon cancer, lung cancer (+smoker)

Multiple myeloma:
normocytic anaemia, check RDW, iron studies,
SPEP
UPEP
Skeletal survey
B2 microglobulin
Albumin: prognostic

INVESTIGATIONS
FBC:
oHg
oMCV (mean corpuscular volume inform morphologic classification)
oMCH (mean corpuscular Hg)
oWCC
infection, inflammation anaemia of chronic dzs
Marrow failure: myeloproliferative, myelodysplasia, leukaemia)
opL (marrow functioning, pL low (DIC), pL high (inflammation & bleeding)
oReticulocytes: immature RCC - marrow functioning (low in marro wfailure, high in infection or
inflammation to fight them)
oHaematocrit: ratio of RBC in plasma and serum - dilution in pregnancy
oRDW increase (combine macrocytic and microcytic)
o
Blood film
Fe studies: Fe, TIBC, Ferritin (sensitive not specific: high (inflammation or infection)), Transferrin
RDW (red cell width distribution): dimorphic anaemia (2 morphologies of RBC: ex both megaloblastic and
microcytic can results in normal MCV need to identify the 2 different causes of anaemia)

Fe DEFICIENCY CHRONIC DZS


Fe STUDIES Low Fe Low or N Fe
High TIBC Low or N TIBC
Low transferrin N transferrin (up in haemochromatosis)
Low ferritin N (haemochromatosis) or high ferritin
(inflammatory condition, infection, alcohol intake)
TX Tx cause
Oral iron
Ferum sulphate

KINETIC CLASSIFICATION
Normal marrow function requires:
Marrow
oFe
oB12
oFolate
Intact blood vessels
Red cell count - protected iron store

MECHANISM CAUSES
Under production Marrow failure
Myeloproliferative
Myelodysplastic
Leukaemia
Aplastic anaemia: Parvovirus B19 infection (slapped cheek syndrome)
Chemotherapy (cytotoxic!)
Meds: Valproate, phenytoin, methotrexate
Nutritional deficiencies
Low EPO
Chronic RF
Tx: give EPO, check serial Hg (stop when Hg normalised prevent hyperviscosity
syndrome, VTE, polycythaemia)
Loss Menstruation
Bleeding
GI malignancy
Hookworm infection in the tropics
Destruction Sickle cell disease/ haemolysis
G6PD: Meds with -sulfa, fava beans
Sideroblastic

Multiple myeloma
Calcium: RANK ligand produced increases osteoclastic acitvity hypercalcaemia (moans- psychiatric:
delirius, confused, depressed, groans, bone, stones- renal calculi, constipation)
Renal diseases: Light chain Ig affects the tubules, amyloid deposition, renal calculi, nephrotoxic chemoT
(Mortezomib: perpheral neuropathy), thalidomide, steroid,
Anaemia
Bone pain: most common presentation, or pathological fractures or bone infection

MX
BONE PAIN HYPERCALCAEMIA ANAEMIA
Skeletal survey: IV biphosphonates: zalendronate, pamidronate Oral (metallic taste,
oXray of long bones and skull Fluids unpleasant) or IV Fe
(lytic lesion: pepper pot skull) Diuretics: Loop (thiazides cause supplements
Bone scan hypercalcaemia) (Galfer-ferrous)

2. A 35 year old woman presents to her General Practitioner complaining of exertional dyspnoea.
When asked to quantify this she states she was previously able to run 5km daily without
difficulty, however she is now finding that she tires much more easily. On review of systems she
mentions that she also commenced a new vegan diet 6 months ago.

MCV: 105
Microcytic megaloblastic (B12, folate, chemotherapy)
Non-megaloblastic (thyroid, alcohol, liver dz)

DDX
Diet: Fe, folate deficiency
Menorrhagia
Anorexia

3. An 82 year old man is referred to the Emergency Department by his General Practitioner. He
complains of fatigue and dizziness on a background of 10kg weight loss over the last 3 months. He
attributes this to a loss of appetite, particularly for meat. Past medical history includes Type 2 Diabetes
and a 40 pack year smoking history. On examination he is pale.

Blood tests reveal a Haemoglobin of 6.7 g/dL (11.5 16g/dL).


Chronic: so not a must to do blood transfusion
MCV 76 (80-Normal)
Iron studies: fe low, tibc high, transferrin sat low, fe3+ low, rdw low

Gi bleed- male
- Upper gi symptom: malaena ===> benign: PUD, malignant: gastric/oesophageal cancer
- Lower gi symptom: BH, blood ====> benign: diverticular dz, AV malformation, malignant: colorectal
cancer

FOBT: very sensitive but not specific


OGD and colonosco

Gynae- women
Haemolysis: usually microcytic due to retics
Screening test for increase cell turnover-check bilirubin, LDH, haploglobin bind to haem and removed so its
going to be low

Causes: autoimmune, sepsis---> MAHA, DIC

Coeliac- anti TTG, dx: bx D2

DDX
GI malignancy (stomach: iron absorption)
Oesophageal malignancy
Colon malignancy
Bronchial malignancy: smoking hx
Monday, 12/10/2015
25.INFLAMMATORY BOWEL DISEASE

CASES
1. A 21 year old lady presents to the Emergency Department complaining of severe crampy abdominal
pain with bloody diarrheoa >10 episodes in the last 24hrs.

Associated symptoms
Crampy abd pain
Tenesmus (UC > likely due to rectal involvement)
Diarrhea: define, volume, colour, clots, mixed, nocturnal, bloody? (> in UC) Mucosy? Mixed with stools or on
toilet paper?
Vomiting: late features perforations
Weight loss
Low grade fever intermittently during flares
Anaemia: lightheadedness, palpitations, fatigue
Extraintestinal manisfestations
o Red eye, discharge (uveitis, episcleritis, conjuctivitis)
o Erythema nodosum
o Pyoderma gangrenosum
o Ulcerative ulcers (> in Crohns)
o Joint pain (monoarthritis, large joint arthritis (ank sponds))
Smoking: protective in UC, RF in Crohns
PSC (> in UC): cholestatic symptoms
Syncope/ presyncope
Urgency
Tensesmus
Post-prandial pain

Physical exam:
Hands: clubbing, anaemia
Abdo exam: tenderness, distension, mass, stoma scars
Extramanifestation: legs
perianal

Exam
Abscess
Fissures
Fistulas (> in Crohns): entero-entero, enterovesical, enterovaginal
Blood PR + FOB

DDX
Acute exacerbation of IBD
Indeterminate: features of both UC & Crohns
Infectious colitis: fever, anyone else with similar presentation, food
Ischaemic colitis:
Occlusive Non-occlusive
Emboli Shock, hypotensive
Diverticulitis
Coeliac disease

INV
FBC: Low Hg (Fe deficiency, normochromic normocytic), high WCC (infectious or chronic inflammatory), low
pL (exclude primary bleeding disorder)
CRP: High (not specific)
U&E: dehydration, baseline renal fx, electrolytes disturbances (hyperchloraemia, hypernatraemia,
hypomagnesemia, hypophosphatemia)
Coags: exclude bleeding disorder
LFTS: Albumin (a measure of nutritional status, liver disease)
Stool culture: C&S, C.Diff, FOB (not requested if already gross bleeding)
Stool chart: frequency of going to the toilet

Colonoscopy: not full scope (risk of perforations), do rigid sigmoidoscopy +/- biopsy
Crohns UC
Crypt abscesses
Skipped lesions
Granulomas
Terminal ileum disease
Fistulas
Strictures
Cobblestone
Transmural inflammation
Consent:
oRF: perforation (in acute inflammation: 1%)
oSedation, not full anaesthetics
PFA: toxic megacolon: >6cm + loss of haustration
Erect CXR: perforation
MRE
Capsule endoscopy

Mx
IV rehydration: bolus + maintenance (3L)
NPO: bowel rest
NG tubes: only in obstructed patients (Crohns from strictures)
Steroids
o IV hydrocortisone 100mg QDS
taper when in remission due to Ses
Cushings: easy bruising + buffalo humps, osteoporosis or osteopaenia (do DEXA scan), weight
gain + increased appetite, diabetes, immunosuppression, mood disturbances
Ab: metronidazole
Anti-pyrexial
o Paracetamol PO 500mg
<45kg: risk of liver failures, tx as child
o 1-3g in low-risk patients
Heparin
o LMWH prophylactic 40mg subcut OD, reduced to 20mg in elderly
o But bleeding risk? Still, massive VTE is > likely
5-ASA
osulfasalazine
omesalazine: given as per rectal enemas (> useful in UC can access rectum)
Immunosuppressants
oMethotrexate: in young pts teratogenic (advise COCP use), thyroid toxicity, BM suppression
oAzathioprine
o6-mercaptopurine
Biological therapy
o Infliximab (anti TNF monoclonal Ab)
o for aggressive control of inflammation (esp UC high likelihood or malignancy)
o if has >2 recurrences per year
o enquire hx of TB (do Mantoux test, hx taking pf previous hx of TB -may reactivate latent TB)
o enquire hx of Hep B too
***Steroids + 5-ASA failed immunosuppressants failed biological therapy

2. A 28 year old gentleman is referred to the Gastroenterology OPD by his GP with a history of bloody
diarrhoea and weight loss of 4kgs over the last year, his past medical history includes an episode of
episcleritis last month.

DDX
Chronic IBD

26.MALABSORPTION

APPROACH
Aetiology
Luminal Enzyme deficiency
oPancreatic enzyme pancreatitis, pancreatic ca, pancreatic insufficiency (malabsorption +
DM), hemi-pancreatectomy
Enzyme malfunction
oPost-Bill Roth surgery (done for gastric ca & previously done for PUD)
Excessive enzyme
oZollinger-Ellison syndrome: high acid release
Bile salts
oReduced production
Chirrosis (fat-malabsorption),chronic hepatitis
oObstruction
biliary stasis (chronic cholangitis, chronic cholelithiasis)
Slow bowel transit, often in diabetes, neuromuscular disease
oPoor absorption
Small bowel bacteria: fat, fat soluble vitamins, B12, folate absorption impaired
Crohns disease
Absorptive/
processing
impairment
Nutritional Coeliac disease: affect D2 part
deficiency Whipples disease
Lymphangitis
Maltoma

Work-Up

LOCALISED CAUSE
Steatorrhoea PANCREAS
Stool culture for C&S: infective causes
Quantity stool test: Calculate fat levels in stools: 24-72hour stool collection (>6g/24hours is
abnormal) but infectious diarrhea can also cause this history!
Quality stool tests: Sudan III staining, steatocrit, triclogin BT?
Amylase: serum
Lipase: serum & stool levels
OGTT: endocrine & exocrine pancreatic insufficiency
Pancreatic enzyme (functional tests): blood glucose levels drop to 1 if has insufficiency not
safe!
CT: calcification + atrophy of pancreas + fat stranding & fluids if acute

LIVER
LFTs
oALT, AST, GGT, ALP [inform on hepatitis not liver fx]
obilirubin, INR, albumin [inform on liver fx, to avoid invasive test as in biopsy below]
Imaging: shrunken atrophic liver
Biopsy: chirrosis ***check INR & Vit K first

BILIARY TRACT
US
ERCP

GLOBAL CAUSE
Coeliac Diagnostic
OGD + D2 biopsy: subtotal villous atrophy
Anti-TTG & anti-endomysial Ab
Other
FBC: Hg Coagulation screen
Blood smears Albumin liver: mucosal damage cause loss of
Fe levels albumin
Folate (red cells) & B12 levels (serum) LDL: hypocholesterolamic, same cause as above
ADEK levels INR

**If first 2 tests N but still suspicious:


Look for other possible causes
Do endoscopy to dx Crohns
o Granulomas, skipped lesions, cobblestone, transmural inflammation
CF Stool-fat tests
Image of biliary tests, pancreas

Amylase, lipase
D-Xylose testing: Look for cause of mucosal damage
Urinalysis: radio-labeled D-Xylose amount in urine
Schilling tests
o give radiolabelled B12 PO, an hour later give IM injection of unlabeled B12 radiolabelled B12
should be absorbed measure amount excreted in urine after 24hours to assess absorption
low(<10%) if diseased
o give intrinsic factors corrected dx B12 deficiency (pernicious anaemia)
o repeat Schilling test + Ab corrected dx small bowel bacterial overgrowth
o Schilling test + pancreatic enzymes corrected pancreatic insufficiency

MX
Calculate caloric intake
Put patient on high calorie high fat diet
Perenteral feeding depending on how much weight lost
Tx underlying cause
COELIAC CROHNS PANCREATITIS
Gluten-free diet + vit, Fe & Steroids, immunosuppressants, 5-ASAs Creon: pancreatic enzyme
folate supplementation Supplementations Supplementations
INFECTIOUS BILIARY DISEASE CIRRHOSIS
Ab Bile acids TIPPS + liver transplant
Campylobacter o Cholestyramine (bile acids sequestrants)
Anti-fungals Stents
Amoebiasis o Cholangiocarcinoma
Giardiasis Remove obstructing lesions - stones

CASES
1. A 35 year old man presents to the Emergency Department on Christmas Day complaining of dull epigastric pain
radiating through to the back, with associated nausea and vomiting. On further questioning he reports that he has suffered
from similar pain intermittently over the last 12 months but it has not been associated with nausea and vomiting in the
past. Review of systems reveals weight loss but the patient is unable to quantify this other than that his belt is now too big.
Past history includes cholecystectomy.He has a 20 pack year history of smoking and estimates his alcohol consumption at
2 bottles/day.

2.A 29 year old woman is referred to the Gastroenterology clinic by General Practitioner. She has been suffering from
crampy abdominal pain over the last year and this occurs during the day, there is no correlation with meal times.
She complains of diarrhoea and constipation alternatively, with no history of weight loss. Past history includes microcytic
anaemia, diagnosed 6 months ago during her first pregnancy.

3.A 24 year old man is referred to the Gastroenterology department by his General Practitioner on a background history of
abdominal pain, diarrhoea and low fever (37.3) over the last 2 months. On review of systems he mentions that he has
noticed stiffness in his knees and lower back over the last 3 weeks. He has no past medical history, no known drug
allergies, with a 1 pack year history of smoking.
Friday, 16/10/2015
27.JAUNDICE

CASES
1. A 44 year old dentist presents to your GP clinic complaining of and a one year history of increasing
fatigue and joint pains, he has recently been told by his family that he looks jaundiced. On exam you
can appreciate that his skin is now darker and note a random blood sugar level of 15. You perform some
screening blood tests which reveal a HbA1c of 9 and a serum ferritin level of 1,500, CRP 5.

DDX
Haemochromatosis
o Jaundice (liver chirrosis), bronze diabetes, elevated ferritin (not specific), joint pains (small facets- hands)
o CRP 5 (normal) truly raised ferritin, no inflammation

DX
Iron study
o Transferrin >45
Genetic testing: C282Y, H63D gene
LFT
Coags
Liver MRI: detect Fe deposition in liver

MX
Surgery
o Liver biopsy (chirrosis)
Phlebotomy

DX OF DM
Fasting glucose >7 at least 8hours
HbA1c
Random glucose
OGTT> 11.2
Hx: when dxd, on meds or insulin, how long on meds, sugar levels, how often check, any hypoglycaemic
episodes, DKA, problem with eye, kidney and feet as a result of diabetes

Complications
Chirrosis
Hepatocellular carcinoma (in 30% of patients with chirrosis)
Renal disease: hepatorenal syndrome
Bronze diabetes
Bone loss: DEXA scan (-2.5)
Cardiac: conduction problem, CCF ECG
Arthropathy
Hyogonadism: Fe deposition on pituitary FSH LH testosterone levels
Hypothyridism: due to reason above

2. A 55 year old lady presents to the emergency department with a two day history of abdominal pain
and swelling. She does not have a GP but reports that her last admission was 3 years ago when she
similarly turned yellow, she estimates her alcohol intake at 2 bottles of wine a day for ages. On
examination you note that she is jaundiced, temperature is 38.5C, the abdomen is distended, diffusely
tender and dull to percussion.

DDX
Decompensated alcohol chirrosis secondary to alcohol use

PE
Hands/ arms
oLeukonycia, clubbing, koilonychia, anaemia (malnourished, B12 deficiency), palmar erythema, spider
naevi, Terrys nail, dupuytrens contracture, hepatic flap
oPulse, BP
oScratch marks, bruising, muscle wasting, tattoos, IV injection marks,
Face
oScleral icterus, fetor hepaticus, poor dental hygiene,
Chest
oGynaecomastia
Abdomen
oAscites, caput medusa (portal hypertension), bruising, hepatosplenomegaly, testicular atrophy

COMPLICATION
SBP

MX
ABC
Ab:
o In first hour of presentation increase survival, MR increase by 7% every 1hour
o Cefuroxime
If septic;
IV fluids
Ascitic tap
Diagnost Therapeut
ic ic
100ml >5L

PHARMACOLOGICAL NON-PHARMACOLOGICAL
Chlorzodiapoxide Dietician assessment: high calorie diet
Alcohol withdrawal Alcohol cessation
Based on CIWA scale Vaccination : Hep B, C, influenza
IV pabrinex Psychiatric, social worker input
Anti-pyrexial Organise surveillance liver US to monitor
Avoid paracetamol for heaptocellular carcinoma
Varices
PPI
Endoscopic banding
Bleeding
Tranexamic acid [NOOOO] Pro-clotting
Vitamin K
Hepatic encephalopathy
Due to increased ammonia from impaired liver
metabolisation or due to brain oedema
Do CT scan
Lactulose and rifampicin to cause diarrhea remove
ammonia
Signs: Increased confusion, altered mental state or GCS
Aetiology: bleeding, infections, anything that induce
hypokalaemia, post-TIPSS
28.CONNECTIVE TISSUE DISEASE

APPROACH
Common clinical presentations
SKIN RASHES SKIN ABNORMALITIES
Livido reticularis [SLE, ssociated with PAN]: Sclerodactyly
purple marbling of skin Thickening and tightening of skin
Photosensitive rash: shawl distribution Associated with systemic sclerosis/
Malar rash scleroderma and CREST syndrome
Eryhthema nodosum Discoid rash
Erythema multiforme: macular or papular, Oral ulcer
red, discrete or indiscrete (multi forms!) Photosensitive
Pyoderma gangrenosum Arthritis
Discoid lupus: red rounded plaquey scaly rash Malar rash
on forehead and scalp (> common in black Immunologic: anti-Sm-antidsDNA, anti-Ro, Anti-
females) LA
***Lupus pernio: large big red rash on nose Neuorologic changes
(associated with sarcoid, not CTDs) ESR
Heliotrophic rash: red purple eyelids,
associated with polymyositis and Renal dzs
dermatomyositis ANA +ve
Gottrons papules: red papules over Serositis: pleuritis
Haematologic disease: haemolytic anaemia, TP,
Psoriasis
LP
JOINTS Clinical presentations
Arthralgia: painful joints without underlying arthritis
Arthritis

Distinguishing features characteristic of CTD (RA etc) arthritis VS degenerative arthritis (OA)
oMorning oPersistent
stiffness o>6/52
oSymmetrical oRheumatoid factor positive
oBilateral oCharacteristic Xray features
oSmall joints
MUSCLES Diseases Clinical presentations
Dermatomyositis Myalgia Proximal muscles myopathy
Polymyositis Myopathy Waddling gait
Muscle wasting
CARDIAC Diseases Clinical presentations
Myocarditis Chest pain
Pericarditis Palpitations
Aseptic endocarditis Dyspnoea
o Libman-sack endocarditis if coexists with SLE
EYES Clinical presentations
Scleritis: painful Keratoconjuctiva sicca: dry eyes
Episcleritis: non-painful Scleromalacia: thin sclera
Iritis/ uveitis: painful red eye Scleromalacia perforans: thin sclera has now perforated
Keratoconjuctivitis: keratin-filled, Sicca complex: systemic illness
decreased acuity Sjogrens syndrome: drying and fibrosis of glands VS
Conjuctivitis xeropthalmia, xerostomia syndrome
LUNGS Diseases Clinical presentations
IPF Pulmonary nodules Dry cough and dyspnoea
o Systemic sclerosis o RA with exertion
o Died from RA Pulmonary vasculitis
Pleuritic inflammation o PE, haemoptysis
KIDNEY Diseases
Lupus nephritis
o SLE
Systemic sclerosis
NEUROLOGICAL SLE GCA
Vasospastic migraine, seizures (if + panvasculitis) Headache, jaw claudication, scalp
In pregnant women tenderness
o Child with conduction abnormality Preceeding proximal myopathy due
o Anti-Ro and Anti-La antibody to polymyalgia rheumatica
HAEMATOLOGICAL Hg, pL, WCC abnormalities
Examples of CTDs
SLE
o Fever + skin, muscle and joint features
Dermatomyositis
o Associated with underlying malignancy do CT thorax and mammogram
Systemic sclerosis
CREST syndrome
o Calcinosis, Raynauds; phenomena, oesophageal dysmotility, sclerodactyly, tealangiectasia

INVESTIGATIONS
DIAGNOSTIC IMAGING OTHERS
FBC Arthroscopy Muscle biopsy:
o Anaemia, leukopaenia (lupus), leukocytosis (inflammation), Joint Xray dermatomyositis
pancytopaenia (1o disease effect or 2o due to meds) o Loss of joint space EEG: seizures
o Low C3, C4 (flare of illness- diagnostic and to monitor disease o Periarticular MSU
progression uptitrate steroids when high) osteopaenia U&E and eGFR:
Auto-antibodies o Chondrocalcinosis renal disease
o Rheumatoid fact, anti-CCP (RA) o Sclerosing cysts PFTs
o Anti-dsNA: lupus CT brain Opthalmology
o Anti-histone: drug-induced lupus CXR
o Anti-smith: primary lupus ECHO
o Anti SCL, anticentromere: systemic sclerosis
o Anti Jo-1 & anti Mi-2: dermatomyositis
o Anti-Ro & anti-La: Sjogrens
Genetic
o HLAs

MX
RA LUPUS SYSTEMIC SCLEROSIS DERMATOMYOSITIS GCA
Pain relief Steroids Steroids & immunomodulators Physio High dose IV
DMARDs o Cyclo- Raynauds: warm cloths & gloves Steroids steroids
Physios phosphamide Erythromycin: promote Immuno- taper
Biological agents o sulfasalazine esophageal motility modulators afterwards
Joint replacement B12 & folate supplements
PROGNOSIS
Disease activity scale (DAS-28)
o >5: poor prognosis

CASES
1.A 20 year old female presents to the Rheumatology clinic complaining of a 4 month history of pain and stiffness in the
MCPs and wrists. The symptoms are worse in the morning lasting for 1 hour and improve with movement. There is fatigue
but no other problems identified on review of systems. On exam the wrists are slightly swollen and tender and MCPs are
tender but not swollen. ESR is 48. The rheumatoid factor (RF) is positive at 105 and the anti-cyclic citrullinated peptide
(anti-CCP or ACPA) is >250. X-rays show peri-articular osteopenia but no erosions.

2.An active 73 year old male presents to the Rheumatology clinic complaining of myalgias, stiffness, wrist and ankle pain
and swelling, fatigue and weight loss. His ESR is 60 and CRP is 9.1. Patient did very well on 20mg/day of prednisone and
this was tapered slowly beginning 3 months after control of his symptoms.
However, symptoms recur, although the ESR and CRP remain normal. The patient was treated with increased doses of
prednisone and then one month after control of symptoms his dose was re-tapered to < 3 mg/day for 6 months before
stopping, approximately 30 months after original diagnosis of PMR. The patient did well for 3 months and then presented
with difficulty in chewing food and fleeting pain in upper extremities. There were no other symptoms and the ESR and
CRP remained normal on no medications.

3.A 54-year-old male presents to his GP complaining of severe pain in his left wrist and right great toe. The affected joints
are swollen, erythematous and exquisitely painful. He denies injury to these areas as well as fever. The patient is afebrile
with a temperature of 37.2, all vital signs are within normal limits. The patient has a history of hypertension and
hyperlipidemia. He has been steadily gaining weight over the past few years and is now about 20 kg overweight. He states
that he drinks four to five beers each night.

On exam, you find that the patients wrist and first MTP joint are erythematous and very tender to even light touch. The
patient has limited range of motion of these joints related to pain. Sensation and capillary refill are intact. Lab values show
a WBC of 5,400 (Normal 4,300-10,800) and a Uric Acid level of 9.7 (Normal >7.0).
Monday, 19/10/2015
29.PROTEINURIA/HAEMATURIA

PROTEINURIA
Definition
Normal proteinuria:<150mg/day
Nephrotic range:>3.5g/day

Dx
Dipstick: Able to detect with large proteinuria, can miss hypoalbuminuria
24 -hours urine collection: GOLD STANDARD
o Acidified: stones collection
o Non-acidified: protein collection
o All voids except first void (bladder stagnant over night bacterial count physiologically high)
o Diurnal variation: orthostatic proteinuria
M, C+S
Protein:Creatinine ratio (PCR): based on spot urine sample
Albumin:Creatinine ratio (ACR): in Diabetics

Aetiology
NEPHROTIC NEPHRITIC
Oedema, low albumin, pro-thrombotic (renal vein Haematuria, light proteinuria, mild hypertension,
thrombosis), hyperchlolesterolaemia (TChol) uraemia, azotemia, oliguria
Minimal change disease Post-Streptococcal Glomerulonephritis
No immune complex deposition IgA nephropathy (> likely to be nephritic)
No infiltration of BM SLE
Good response to steroid Immun eocmplex deposition
Excellent prognosis o IE
o ANCA-positive vasculitis
FSGS (Focal sclerosing Glomerular Sclerosis) Cryoglobulinaemia, PAN, HUS
Chronic inflammatory process o Goodpasteurs syndrome (Anti-GBM)
o Reflux nephropathy scarring Alport-s syndrome
Heroin drug users o Hearing loss and renal disease
HIV HSP
o HIV-associated Nephropathy (HIVAN) o > common in children
o Increased risk of death Hereditary GN
o Usually get the collapsing subtype
Familial

SLE
Membranous nephropathy
IgA nephropathy
Amyloidosis
AL: light chain associated with multiple myeloma

Investigations
BLOODS NON-BLOODS
FBC U&E
Hg: anaemia [CKD] EPO production Hyperkalaemia
ESR & CRP Cr
Auto-antibodies eGFR
ANA Cockcroft-Gault, MDRD (Modification of Diet in Renal
AMA: PBC Disease), CKD-EPI score
Anti-dsDNA: Lupus Age, race, ethnicity, serum Cr
anti-Ro, anti-La: Sjogrens C&S
antiSmAb: Lupus
anti-GBM: Goodpasteurs syndrome
c-ANCA: Wegeners [granulomatosis with polyangiitis]
p-ANCA: Churg-Strauss, MPA
ASOT: nephritic syndrome
Cryoglubulin: cryoglobulinaemia
Viral serology: HIV, Hep B & C

CKD
STAGES eGFR
Stage I >90
Stage II 60-89
Stage III 3a: 45-59 3b: 31-44
Stage IV 15-30
Stage V <15

Biopsy
Indication Contraindication
When there is no obvious cause of proteinuria (unknown Small kidneys
aetiology) o Increased risk of damage to vessels
Rapidly deteriorating renal function Uncontrolled bleeding
Lack of response to medication o Coagulopathy
o On anti-coagulation medications

CASES
1. A 50 year old gentleman presents to his GP complaining of puffiness of his face. On review of systems he
reports frothy urine for the last week. Physical examination reveals pitting oedema to the knees bilaterally,
oedematous hands and face. He has no past medical history. Urine dipstick reveals heavy proteinuria.
DDX
Nephrotic syndrome
Transient proteinuria: UTIs, exercises, orthostatic
Diabetes: diabetic nephropathy
HTN: Check BP, ECG (L ventricular hypertrophy), urine (microalbuminuria)
Malignancy: multiple myeloma (monoclonal expansion of protein bands + decrease in albumin)

2. A 33 year old gentleman is referred to the Nephrology clinic when a visa application medical exam reveals
painless microscopic haematuria. On review of systems he reports he is falling behind at work due to excessive
tiredness. Urine dipstick confirms haematuria.
DDX
Glomerulonephritis
Dx
Dipstick
Haematuria, leucocytosis, nitrites presence, glucose & ketones in diabetics
MACROSCOPIC/ FRANK HAEMATURIA
PRESENTATION Painful
AETIOLOGY Renal calculi
Malignancy
o renal cell carcinoma, TCC (renal pelvis, ureter, especially bladder), squamous cell cacrcinoma
Polycystic kidney
o Bleed when trauma on cysts haemorrhagic cysts
GN
o capillary necrosis AKI nephritic syndrome
Severe infection
o UTI, pyelonephritis
Beets, rifampicin

Follow-up Mx
Monitor renal function Dialysis preparation
If deteriorating: check nephrotoxic medications AV fistula formation, vascular surgeons
o NSAIDS, aminoglycosides (gentamicin), ACE-I, ARBs, metformin involvement, surgical preparation
Check BP Central venous access: PermaCath
Check signs of oedema Loop diuretics: Furosemide
Renal diet: low salt, low potassium, low phosphate ( low Vit D, low ACE-I
Ca, high PO4)
Check PTH Types
1y: Raised PTH, raised Ca Peritoneal dialysis
2:Raised PTH, normal Ca Well-organised
Sterile procedure patent must be well-
If high: Give Vit D educated
Fails Phosphate binders (a form of Ca: Ca carbonate or chloride) Must have some rrenal fx preserved
Fails Ca mimetics (Cinecalcet)
30.CARE OF THE PALLIATIVE PATIENT
APPROACH
Common palliative medical conditions
Cancer Degenerative: MS, MND, strokes
PF ESRD, end-staged liver disease
COPD Diabetes

Common clinical problems


PRESENTATIONS AETIOLOGY
PAIN Types Aetiology Treatment
Neuropathic Cancer Neuropathic pain
MS Pregabalin
Drugs SEs Gabapentin
TCA: amitryptilline
Anti-epileptics
SSRI: not as good as TCA
Steroids: dexamethasone
Nerve block, epidurals
TENS machine
Nerve stimulator
Muscle relaxants
o Baclofen (PO or intrathecal), BOTOX,
Levetericetam
Bone Mets Bony pain
Fractures NSAIDS
Metabolic abnormality IV biphosphonate (hypercalcaemia):
(hypercalcaemia) in malignancy zalendronate, given with lots of fluids
& CKD (hyperPTH) Radiation
Visceral Malignancy: solid organ
tumour compression

MX
Principles
By mouth, by time, by ladder: PO, regular (prevent nociceptors increased sensitivity to pain),
according to pain ladder
WHO pain ladder
Paracetamol: IV
NSAIDs: topical, per rectum
Weak opioids: IV, transdermal
Strong opioids
***If chronic, bypass ladder
***Know change in route of administration (fentanyl)
CONSTIPATION Additional medications for managing SEs
Medications Aetiology Examples
PPI Paracetamol, NSAIDs Omeprazole 100mg
Gastritis Lansoprazole
Anti- Opioids Lactulose 10-15mls, TDS/QDS: treat hardening of
constipation Constipation from anaemia stools
and hypercalcaemia Senna (2nd): stimulates bowel movements
Obstruction Pro-kinetics: promotes gastric emptying, CI in
Laxatives + stenting and bowel obstruction
radiotherapy Metoclopramide, erythromycin
Fluids IV/ SC
N&V Medications Aetiology Examples
Anti-emetics Chemotherapy CNS emesis
Electrolytes imbalance: Ondansetron 8mg TDS
hyponatraemia, Cyclizine 10-100mg TDS
hypercalcaemia Local causes
Biliary stasis Domperidone 10mg TDS
Uraemia Metoclopromide 10mg TDS
Erythromycin
Steroids: dexamethasone
Haloperidols
DEHYDRATION Aetiology Tx
Bowel/GI Anti-diarrheal
Short bowel syndrome Loperamide 2mg TDS
High-output ileostomy
Large-output diarrhea
Infection
Chemotherapy complications Optimise gut health and rehydrate

Mx
FLUID
Calculate fluid requirements Type of fluids
100ml for first 10kg Normal saline, crystalloid SC (max 1L/hour)
50mls for next 10kg
25mls thereafter
Ex: 50kg = 2250mls
SOB Aetiology Tx
Primary lung disease Anti-cholinergics: Bosropan (abdominal pain): dry
Lungs or mediastinum mets up respiratory secretions: used in end-stage
Neurodegenerative diseases (MND, etc) Radiotherapy: for mets
Nerve involvement: vagus, phrenic O2 supports: BiPAP, CPAP
Sitting and sleeping patient upright
Physiotherapy
Haloperidol: for respiratory distress
Codeine
WEIGHT LOSS Aetiology Tx
Anorexia Nutritional review
Local problems compression of Diet: high protein high fat diet
gastric wall : early satiety Parenteral feeding: require close nursing care,
Chemotherapy worsens constipation and N&V
Increased effort of metabolism (higher Increase appetite: methylphenidates, megestrol
caloric needs) acetate [Megace] + dexamethasone
Severe respiratory distress
Psychological
ANOREXIA As above
SKIN Decubitus/ pressure ulcer
Tx
Changing positions
Washouts
Ammonium therapy
Cavilon spray: gives extra layer of protection of sore
Specific dressings (hydrocolloid or foam): involve wound nurse
PSYCHOLOGICAL Examples Aetiology
Depression Meds: opioids
Delirium Infection
Constipation
Electrolyte abnormality
Metabolic abnormality
Dementia (in neurodegenerative disease)

Mx
Treat underlying cause
Stable and familiar environment promote orientation
1:1 nursing in an isolated room
Fails haloperidol, risperidone rarely used due to reisperidone-induced neutropaenia SE

PALLIATIVE
Actively dying
Timeframe? Depends on
2-3days: IV Abs et cnot appropriate
Longer: Relenvant to treat

CASES
1. A 41 year old man attends the oncology outpatients. He has recently been diagnosed with a metastatic colon
carcinoma. He started strong opioids 5 days ago and has been admitted complaining of nausea and
abdominal discomfort. On examination he is pale and appears clinically dehydrated.

2. A 74 year old man is admitted to hospital. He has known prostate carcinoma with bone metastases. Despite
this, he is complaining of a lot of pain in his lower back where he has known lumbar spine bony metastases.
The pain can come on quite suddenly and can last for hours. He is on regular oral ibuprofen 400mg three
times daily and is taking PRN oral immediate-release morphine 10mg about twice a day.

3. A 68 year old man attends the respiratory outpatients. He was diagnosed with COPD six years ago. He feels
that his condition is severely limiting his quality of life and activity, and complains of severe shortness of
breath on minimal exertion, worse in the last few weeks, and is largely immobile as a result. He has had
three admissions to hospital in the last 5 months with acute exacerbations, and is reluctant to go to A&E in
the event of another episode. He is anxious about his future care options. He also feels he cannot talk to his
wife and family about the future without upsetting them.

Você também pode gostar